You are on page 1of 112

NBME FORM YEAH YEA 2

Block 1

1.) A 50-year-old man with a 20-year history of type


2 diabetes mellitus has had sensory neuropathy for 2 weeks.
Pulses are decreased at the ankle. There is no peripheral edema. His serum
glucose levels have been between 150 mg/dL and 200 mg/dL over the past 6
months. Which of the following is the most effective measure to prevent
serious foot infections?
A) Use of support hose
B) Use of well-fitted shoes
C) Prophylactic antibiotics
D) Daily aspirin and dipyridamole
E) Decrease in serum glucose level
2.) A 63-year-old woman comes to the emergency
department because of palpitations for 1 hour. She was diagnosed with
hyperthyroidism 2 months ago but has not received treatment. Three
hours before arrival, she drank two bottles of beer with dinner. Her blood
pressure is 90/60 mm Hg, and pulse is 85/min and irregularly irregular.
Examination shows a variation in intensity of S1. There is an absence of
waves in the venous pulse. An ECG is shown. Which of the following is
the most likely cardiac rhythm disturbance?
A) Atrial fibrillation
B) Atrial flutter
C) Multifocal atrial tachycardia
D) Paroxysmal atrial tachycardia
E) Second-degree atrioventricular block, Wenckebach type
F) Sick sinus syndrome
G) Sinoatrial exit block, type II
H) Torsades de pointes
I) Wandering pacemaker
J) Wolff-Parkinson-White syndrome
3.) A 5-year-old girl is brought to the physician 30 minutes after
being bitten on the forearm by her cat. Examination shows a small
puncture wound covered with dried blood. She is at increased risk of
infection for which of the following reasons?
A) Arm wounds are more susceptible to infection

B) Salivary enzymes in cats augment infection


C) It is difficult to cleanse pathogens from this wound
D) The Staphylococcus species involved is particularly virulent
E) Young children are less able to resist infection than adults
4.) A 4-year-old girl is brought to the physician because of pallor
and jaundice for 2 days. She had previously been well, although she was
treated for jaundice with phototherapy for 2 weeks while a newborn.
Her mother and two additional maternal relatives underwent splenectomy
during childhood for unknown reasons. Examination of the patient shows
jaundice. The spleen tip is palpated 4 cm below the left costal margin.
Which of the following blood smear findings is most likely to explain this
family's condition ?
A) Elliptocytes
B) Howell-Jolly bodies
C) Schistocytes
D) Sickle cells
E) Spherocytes
5.) A sexually active 20-year-old woman has had fever, chills, malaise, and pain of the
vulva for 2 days. Examination shows a vulvar pustule that has ulcerated and
formed multiple satellite lesions. Nodes are palpated in the inguinal and
femoral areas. A smear of fluid from the lesions establishes the diagnosis.
Which of the following is the most likely causal organism?
A) Chlamydia trachomatis
B) Haemophilus ducreyi
C) Neisseria gonorrhoeae
D) Streptococcus pyogenes (group A)
E) Treponema pallidum
6.) A 5-year-old boy is brought to the physician 1 hour after
urinating bright red blood. He has been taking ibuprofen since injuring his
right flank while wrestling with friends yesterday; he also has been
taking penicillin for 3 days for streptococcal pharyngitis. His
temperature is 36.7 C (98 F), blood pressure is 90/48mm Hg, pulse is 108/min,
and respirations are 18/min. Examination shows purple ecchymoses over
the shins and right flank; there is tenderness of the right
costovertebral area. The abdomen is nontender. Genital examination shows no
abnormalities. There is no edema. Urinalysis shows gross blood;
microscopic examination shows 510 leukocytes/hpf and erythrocytes that
are too numerous to count. Which of the following is the most likely
explanation for this patient's hematuria?
A) Acute pyelonephritis
B) Ibuprofen-induced acute papillary necrosis

C) Post-streptococcal glomerulonephritis
D) Rhabdomyolysis
E) Traumatic injury to the kidney
7.) A 16-year-old girl comes to the physician for her first prenatal
visit at 12 weeks' gestation. She has not had any immunizations since
the age of 5 years. She has received the following immunizations at the
recommended ages:
5
3
1
4

Diphtheria-tetanus-pertussis
Hepatitis B
Measles-mumps-rubella
Oral poliovirus

Examination shows no abnormalities. Which of the following is the most


appropriate immunization to administer at this visit?
A) Diphtheria-tetanus-pertussis
B) Diphtheria (child)-tetanus toxoid
C) Diphtheria (adult)-tetanus toxoid
D) Haemophilus influenzae type b
E) Hepatitis B
F) Inactivated poliovirus
8.) A 12-year-old boy is brought to the physician because of
temperatures to 40.2 C (104.4 F), lethargy, and a stiff neck for 2 days. A
diagnosis of pneumococcal meningitis is made, and the child is admitted to
the hospital for antibiotic therapy. Serum studies 24 hours after
admission show:
Na+ 117 mEq/L
Cl 89 mEq/L
HCO3 25 mEq/L
Which of the following is the most likely explanation for these findings?
A) Excessive cortisol release
B) Hyperaldosteronism
C) Inappropriate secretion of ADH (vasopressin)
D) Increased production of leukotriene C4
9. )An asymptomatic 57-year-old man with a 3-year history of type 2
diabetes mellitus comes for a routine follow-up visit. Examination shows no
abnormalities. Serum studies show:
Aspartate aminotransferase (AST, GOT) 76 U/L
Alanine aminotransferase (ALT, GPT) 86 U/L
Iron 260 g/dL Total iron-binding capacity 300 g/dL (N=250450)
Ferritin 1200 ng/mL Antinuclear antibody negative

Serologic testing for hepatitis is negative. Which of the following is


the most appropriate next step in management?
A) Corticosteroid therapy
B) Interferon therapy
C) Penicillamine therapy
D) Chronic phlebotomy
E) No therapy indicated
10.) A 26-year-old man has had anxiety and insomnia since he lost his
job 1 month ago. He also has had palpitations, daily headaches, and
flatulence. He has no history of psychiatric disorders. His mother has
a history of bipolar disorder, mixed, and his father has alcoholism;
his younger sister had a history of drug abuse but has been abstinent
from drugs for 3 years. Which of the following is the most likely
diagnosis?
A) Adjustment disorder
B) Bipolar disorder, depressed
C) Generalized anxiety disorder
D) Major depressive disorder
E) Post-traumatic stress disorder

11.) A previously healthy 32-year-old man comes to the emergency


department because of a 12-hour history of fatigue and shortness of breath
at rest. He takes no medications. His temperature is 7 C (98.6 F), blood pressure is
96/58 mm Hg, pulse is 110/min, and respirations are 22/min. Examination shows
scleral icterus and conjunctival pallor. There is no hepatosplenomegaly. Laboratory
Studies show:
Hemoglobin 6.2 g/dL
Serum Leukocyte count 8800/mm3
Bilirubin, total 8.5 mg/dL
Reticulocyte count 16%
Direct 1.5 mg/dL
Platelet count 245,000/mm3
Lactate dehydrogenase 1260 U/L
A) Acute toxic hepatitis
B) Alcoholic hepatitis
C) Cholangiocarcinoma
D) Chronic idiopathic cirrhosis
E) Common bile duct calculus
F) Congestive hepatitis
G) Gallstone pancreatitis

H) Gilbert's syndrome
I) Infectious hepatitis
J) Intravascular hemolysis
K) Primary biliary cirrhosis
12.) A 24-year-old third-year medical student comes to the student health clinic for
evaluation of jaundice which he noticed this morning. He has had no abdominal
pain, itching, or weight change. He takes no medications. He describes recent
anxiety over anticipation of the upcoming Step 2 examination. His temperature is
37 C (98.6 F), blood pressure is 120/80 mm Hg, pulse is 76/min, and respirations
are 12/min. Examination shows no abnormalities except for scleral icterus.
Laboratory studies show:
Hemoglobin 15 g/dL Serum
Leukocyte count 7000/mm3
Alkaline phosphatase 90U/L
Platelet count 240,000/mm3
Aspartate aminotransferase Serum (AST, GOT) 23 U/L
Bilirubin, total 3.5 mg/dL
Alanine aminotransferase Direct 0.7 mg/dL (ALT, GPT) 27 U/L
A) Acute toxic hepatitis
B) Alcoholic hepatitis
C) Cholangiocarcinoma
D) Chronic idiopathic cirrhosis
E) Common bile duct calculus
F) Congestive hepatitis
G) Gallstone pancreatitis
H) Gilbert's syndrome
I) Infectious hepatitis
J) Intravascular hemolysis
K) Primary biliary cirrhosis
13.) A 37-year-old woman with a 2-year history of hypertension comes for a follow-up
examination; her hypertension has worsened despite treatment with a low-sodium
diet and a -adrenergic blocking agent. She has a history of rheumatic fever and
Graves' disease treated with 131I. Her blood pressure is 160/106 mm Hg, and
pulse is 80/min. Serum studies show:
Na+ 135 mEq/L
Cl 100 mEq/L
K+ 3.4 mEq/L
HCO3 26 mEq/L
Urea nitrogen (BUN) 12 mg/dL
Creatinine 0.8 mg/dL

Urinalysis is within normal limits. Renal ultrasonography shows a left kidney with a
markedly irregular contour; it is 2.8 cm smaller than the right.
A) Adrenal cortex
B) Adrenal medulla
C) Aorta
D) Renal arteries
E) Renal glomeruli
F) Thyroid gland
14.) A 27-year-old woman comes to the physician because of muscle weakness and
cramps for 2 weeks. She has been taking a -adrenergic blocking agent for
hypertension for 2 years. She had chronic lymphocytic thyroiditis (Hashimoto's
disease) 1 year ago. Her blood pressure is 160/108 mm Hg, and pulse is 60/min.
Serum studies show:
Na+ 140 mEq/L
Cl 110 mEq/L
K+ 2.2 mEq/L
HCO3 30 mEq/L
Urea nitrogen (BUN) 20 mg/dL
Creatinine 1 mg/dL
Magnetic resonance angiography of the abdomen shows normal findings.
A) Adrenal cortex
B) Adrenal medulla
C) Aorta
D) Renal arteries
E) Renal glomeruli
F) Thyroid gland
15.) A 24-year-old man is hospitalized for treatment of a posterior dislocation of the
right knee sustained in a motorcycle collision. Six hours after closed reduction,
previously present distal pulses in his foot are absent, but the foot has remained
warm. Which of the following is the most appropriate next step in management?
A) Elevation of the limb and observation for 24hours
B) Nitroprusside therapy
C) Sympathetic block
D) Femoral arteriography
E) Embolectomy with a Fogarty catheter through aproximal arteriotomy
D) Inflammation of the origin of the plantar fascia

16.) A healthy 4-year-old girl is brought for a well-child examination. A grade 2/6
systolic ejection murmur is heard along the upper left sternal border. S2 is widely
split and does not vary with respiration. A soft mid-diastolic murmur is heard along
the lower left sternal border. Examination shows no other abnormalities. Which of
the following is the most likely diagnosis?
A) Aortic stenosis
B) Atrial septal defect
C) Coarctation of the aorta
D) Mitral valve prolapse
E) Patent ductus arteriosus
F) Pulmonary stenosis
G) Tetralogy of Fallot
H) Transposition of the great arteries
I) Ventricular septal defect
J) Normal heart

18.) A previously healthy 57-year-old woman comes to the physician because of three
episodes of blurred vision in the right eye over the past 3 weeks; each episode
lasts approximately 5minutes. Retinal examination shows a small refractile body at
The bifurcation of a retinal artery. The remainder of the examination shows no
abnormalities. Which of the following is the most appropriate next step in
diagnosis?
A) Cerebral angiography
B) Echocardiography
C) Electroencephalography
D) Duplex scan of the carotid arteries
E) MRI of the brain
19.) A 67-year-old woman comes to the physician for her first influenza virus
vaccination. She has a history of untreated hypertension. Her blood pressure is
160/100 mm Hg, and pulse is 100/min. Shortly after administration of the
influenza virus vaccine, she develops shortness of breath, hives, and angioedema.
Which of the following is most likely to have prevented this reaction?
A) Inquiry about an egg allergy
B) Heterophile agglutination test
C) Skin test with histamine reagent

D) -Adrenergic blocking agent therapy


E) Amantadine therapy
F) Insulin therapy
G) Rimantadine therapy

20.) A 7-year-old girl is brought to the physician because of a 2-day history of fever,
headache, sore throat, and swollen glands. She does not have a runny nose,
congestion, or cough. She has no allergies to medications. Her temperature is
38.6C (101.4 F), blood pressure is 100/60 mm Hg, pulse is 120/min, and
respirations are 16/min. Examination shows a swollen, erythematous oropharynx
With tonsillar exudates. The anterior cervical lymph nodes are enlarged and
tender. No other abnormalities are noted. Which of the following is the most likely
causal organism?
A) Adenovirus
B) Corynebacterium diphtheriae
C) Group A streptococcus
D) Haemophilus influenzae
E) Mycoplasma pneumoniae
21.) A 70-year-old nursing home resident is admitted to the hospital because of
progressive obtundation over the past 2 days. He has tachycardia, tachypnea, and
hypotension. Bilateral basilar crackles and an S3 gallop are heard on auscultation.
Examination shows jugular venous distention and peripheral edema. Swan-Ganz
catheterization shows a cardiac index of 1.8 L/min/m2 (N=2.54.2), a mean
Pulmonary capillary wedge pressure of 23 mm Hg (N=110), and markedly
Increased systemic vascular resistance. Which of the following is the most likely
diagnosis?
A) Cardiogenic shock
B) Hypovolemic shock
C) Neurogenic shock
D) Septic shock

22.) A 42-year-old man with alcoholism is brought to the emergency department by a


friend because of fever and progressive shortness of breath for 12 hours. His
friend reports that they were eating leftover chicken and drinking beer earlier in
the day when the patient suddenly choked and vomited. Six hours later, he
developed sweating, chills, and shortness of breath. He has not had any
subsequent nausea, vomiting, hematemesis, or abdominal pain. He has smoked
Two packs of cigarettes daily for 22 years and drinks 12 to 18 beers daily. He is in
moderate respiratory distress. His temperature is 39.3 C (102.8 F), blood pressure
is 90/60 mm Hg, pulse is 120/min, and respirations are 24/min. Examination
shows no jugular venous distention. Breath sounds are decreased halfway up the
left lung with increased dullness. Cardiac examination shows a normal S1 and S2;
no murmurs are heard. There is no abdominal tenderness. Bowel sounds are

hypoactive. X-ray films of the chest show a left pleural effusion and air in the
mediastinum. Thoracentesis is performed.
Laboratory studies show:
Leukocyte count 18,000/mm3
Segmented neutrophils 85%
Bands 10%
Lymphocytes 5%
Serum Protein 6 g/dL
Lactate dehydrogenase 200 U/L
Pleural fluid Leukocyte count 8000/mm3
Segmented neutrophils 98%
Monocytes 2%
Protein 4.2 g/dL
Amylase 140 U/L
Lactate dehydrogenase 180 U/L
Gram's stain
WBC present
Organisms none
Which of the following is the most likely diagnosis?
A) Congestive heart failure
B) Esophageal rupture
C) Pancreatitis
D) Pericarditis
E) Tuberculosis
23.) A 2-year-old girl with tricuspid atresia has increasing respiratory distress for 2
days. She has been recovering uneventfully from an operation 10 days ago to join
systemic venous return with pulmonary arterial circulation. Over the past 4 days,
she has been weaned off mechanical ventilation, started on oral feedings, and
is receiving chest physiotherapy for atelectasis. Her temperature is 37.4 C (99.3 F),
blood pressure is 98/64 mm Hg, pulse is 120/min, and respirations are 46/min.
Examination shows nasal flaring, grunting, and intercostal retractions. An x-ray
film of the chest shows large bilateral pleural effusions. Thoracentesis yields 300
mL of whitish-yellow fluid. The supernatant remains uniformly opaque on
centrifugation. Which of the following is the most likely cause of the pleural
effusions?
A) Chylothorax
B) Congestive heart failure
C) Empyema
D) Pulmonary embolism
E) Superior vena cava obstruction
24.) A 23-year-old man is brought to the physician by his mother because he has heard
a voice over the past month telling him to hurt himself. His mother says that her
son has no friends and is a lifelong loner; since graduating from high school, he
has been unable to hold a job. He admits to smoking marijuana occasionally and
drinking six beers weekly. Examination shows a poorly groomed man with poor
eye contact. He has a flat affect and limited facial expression. He says he has no
intention of harming himself or others. Which of the following is the most

appropriate next step in management?


A) Schedule a follow-up visit in 4 weeks
B) Prescribe oral risperidone and schedule a follow-up visit in 2 weeks
C) Admit him to the partial hospital program and prescribe oral lithium carbonate
D) Admit him to the psychiatric unit for detoxification
E) Admit him to the psychiatric unit and prescribe oral imipramine

25.) A 23-year-old man is brought to the physician by his mother because he has heard
a voice over the past month telling him to hurt himself. His mother says that her
son has no friends and is a lifelong loner; since graduating from high school, he
has been unable to hold a job. He admits to smoking marijuana occasionally and
Drinking six beers weekly. Examination shows a poorly groomed man with poor
Eye contact. He has a flat affect and limited facial expression. He says he
has no intention of harming himself or others. Which of the following is the most
appropriate next step in management?
A) Schedule a follow-up visit in 4 weeks
B) Prescribe oral risperidone and schedule a follow-up visit in 2 weeks
C) Admit him to the partial hospital program and prescribe oral lithium carbonate
D) Admit him to the psychiatric unit for detoxification
E) Admit him to the psychiatric unit and prescribe oral imipramine

26.) An 18-month-old boy is brought for a well-child examination. He was born at 37


weeks' gestation and weighed 2800 g (6 lb 3oz). There were no prenatal or
perinatal complications. Developmental history indicates that he is able to run and
walk up stairs while holding his mother's hand. He can hold a crayon but does not
Scribble spontaneously. He is at the 40th percentile for length and 50th percentile
for weight. Examination shows a crude pincer grasp. Which of the following is the
most appropriate assessment of fine and gross motor development?
Fine Motor Gross Motor
Development Development
A) Delayed normal
B) Normal delayed
C) Delayed delayed
D) Normal normal

27.) Five months after beginning fluoxetine to treat obsessive-compulsive disorder, a


19-year-old man states that he discontinued his medication 2 months ago because
he had begun to worry about taking his medication every day. His initial response

to the medication was good. His symptoms have now returned, and his morning
ritual of cleaning and grooming consumes so much time that his job is in jeopardy.
In addition to education about the nature of his disorder and its treatment, which
of the following is the most appropriate next step in management?
A) Tell the patient to schedule a return visit as needed
B) Offer to change the medication
C) Request that the patient's parents superviseadministration of medication
D) Monitor the patient's compliance by weekly blood tests
E) Begin a trial of cognitive-behavior therapy

28.) A 32-year-old woman comes to the physician because of a 3-month history of


increasing pain and stiffness in her wrists, hands, and ankles. During this period,
she also has had progressive fatigue and morning stiffness lasting 2 hours. She has
a 1-year history of rheumatoid arthritis treated with naproxen. Examination shows
redness, swelling, and warmth over the wrist, hand, and ankle joints bilaterally.
There are nontender subcutaneous nodules over the extensor surfaces of both
elbows. X-ray films of the hands show diffuse osteopenia and erosions over several
of the distal metacarpal bones. Which of the following is the most appropriate
pharmacotherapy?
A) Add oral cyclophosphamide
B) Add oral gold
C) Add oral methotrexate
D) Add oral penicillamine
E ) Switch to oral ibuprofen

28.) A previously healthy 62-year-old man comes to the physician because of a 2-month
history of progressive shortness of breath and a mild nonproductive cough. He
does not smoke. He worked in a foundry most of his adult life before retiring 2
years ago. Vital signs are within normal limits. Crackles are heard at both lung
Bases with no wheezes. Cardiac examination shows an accentuated P2. The
remainder of the examination shows no abnormalities. An x-ray film of the chest
shows prominent interstitial markings at the lung bases. Echocardiography shows
an ejection fraction of 55%. Pulmonary function testing is most likely to show
which of the following?
A) Decreased FEV1:FVC ratio
B) Decreased maximal inspiratory effort
C) Decreased total lung capacity
D) Increased forced vital capacity
E) Normal carbon monoxide diffusion capacity

29.) A 55-year-old man is admitted to the hospital because of progressive shortness of


breath for 10 days. He has a history of chronic venous stasis and deep venous
thrombosis. He has been treated with warfarin since he had several pulmonary
emboli 2 years ago; he takes no other medication. He has smoked two packs of
Cigarettes daily for 30 years. He weighs 109 kg (240 lb) and is 165 cm (65 in) tall.
Examination shows jugular venous distention. Echocardiography is most likely to
show which of the following?
A) Left ventricular dilation
B) Mitral stenosis
C) Pericardial thickening and tamponade
D) Right ventricular hypertrophy and dilation
E) Tricuspid stenosis
30.) A 32-year-old woman comes to the physician because of weakness of the lower
extremities for 2 days. Three years ago, she had pain and partial loss of vision of
the right eye; the vision returned to normal after 6 weeks. There is mild pallor of
the right optic disc. She has impaired tandem gait. Babinski's sign is present
bilaterally. There is mild spasticity of the lower extremities and mild weakness of
the iliopsoas and hamstring muscles. Serum creatine kinase activity is 50 U/L.
Which of the following is the most likely diagnosis?
A) Amyotrophic lateral sclerosis
B) Becker's muscular dystrophy
C) Diffuse sensorimotor peripheral neuropathy
D) Duchenne's muscular dystrophy
E) Hyperkalemic periodic paralysis
F) Hypokalemic periodic paralysis
G) Multiple sclerosis
H) Myasthenia gravis
I) Myasthenic (Lambert-Eaton) syndrome
J) Myotonic muscular dystrophy
K) Polymyositis
L) Pontine astrocytoma
M) Pontine infarction
N) Spinal cord tumor
O) Transverse myelitis

31.) A healthy 42-year-old man comes to the physician for a life insurance evaluation.
He smoked one-half pack of cigarettes daily for 20 years but quit 10 years ago. His

father died of a myocardial infarction at the age of 65 years. The patient weighs 93
kg (205lb) and is 178 cm (70 in) tall. His blood pressure is 160/110 mm Hg,
pulse is 96/min, and respirations are 16/min. Physical examination, ECG, and an xray film of the chest show no abnormalities. Laboratory studies are within normal
limits except for a serum cholesterol level of 206 mg/dL. Which of the following is
the greatest risk factor for cerebral infarction in this patient?
A) Genetic profile
B) History of smoking
C) Hypercholesterolemia
D) Hypertension
E) Obesity
32.) A previously healthy 56-year-old woman comes to the physician because of
jaundice and dark urine for 3 weeks. She has a 1-year history of generalized
pruritus. She takes no medications. Examination shows jaundice and several
ecchymoses over the forearms and thighs. The liver and spleen are enlarged and
nontender. Laboratory studies show:
Prothrombin time 18 sec
Serum Protein Total 8.5 g/dL
Albumin 3.8 g/dL
Bilirubin Total 5 mg/dL
Direct 2 mg/dL
Alkaline phosphatase 150 U/L
Alanine aminotransferase (ALT, GPT) 45 U/L
Antimitochondrial antibody assay is strongly positive.
A CT scan of the abdomen shows hepatosplenomegaly.
Endoscopic retrograde cholangiopancreatography shows no abnormalities.
Because of her condition, this patient is at greatest risk for which of the following
deficiencies?
A) Niacin
B) Vitamin A
C) Vitamin B2 (riboflavin)
D) Vitamin B12 (cyanocobalamin)
E) Vitamin C
33.) An asymptomatic 52-year-old man comes for a
follow-up
examination 1 month after he passed renal calculi. He
has a history of renal
calculi 2 years ago. Serum uric acid and calcium
levels and urinary
oxalate excretion are within normal limits. Urinary
calcium excretion is
increased. In order to avoid recurrence of renal
calculi, which of the
following is the most appropriate pharmacotherapy for

this patient?
A
) Bicarbonate
B
) Calcium lactate
C
) Methenamine mandelate
D
) Probenecid
E
) Thiazide diuretic
34.
A 57-year-old woman is brought to the emergency
department because of
abdominal pain for 12 hours. Over the past 3 hours,
the pain has become
severe and generalized. Over the past month, she has
had mild upper
abdominal discomfort that is relieved by eating. She
has a history of
recurrent migraines treated with sumatriptan as
needed. Her temperature
is 38.2 C (100.8 F), blood pressure is 170/95 mm Hg,
and pulse is
110/min. Abdominal examination shows mild distention;
there is marked
rigidity with diffuse tenderness. Bowel sounds are
absent. Rectal
examination shows no abnormalities; test of the stool
for occult blood is
negative. Laboratory studies show:
Hematocrit 36%
Leukocyte count 16,500/mm3
Serum
Na+ 145 mEq/L
Cl 106 mEq/L
K+ 3.8 mEq/L
HCO3 19 mEq/L
Urea nitrogen (BUN) 32 mg/dL
Which of the following is the most appropriate next
step in diagnosis?
A
) X-ray films of the abdomen while supine and
standing
B
) Abdominal ultrasonography
C
) Upper gastrointestinal series with contrast

D
) HIDA scan
E
) Fiberoptic endoscopy of the upper
gastrointestinal tract

35. An 18-year-old man comes to the physician


because of itchy
lesions on his penis for 2 weeks and itching around
his wrists and ankles
for 1 week. He is sexually active and does not
consistently use condoms.
Examination shows scattered, crusted lesions on the
penis and no
significant inguinal lymphadenopathy. There are
excoriated papules on the
wrists and ankles, a few papules between the fingers,
and excoriations
along the belt line. Which of the following is the
most likely
diagnosis?
A
) Disseminated primary herpes simplex
B
) Eczema
C
) Scabies
D
) Secondary syphilis
E
) Vasculitis
36. An asymptomatic 62-year-old man comes for a
follow-up visit.
One month ago, he had acute cystitis treated with
ciprofloxacin. At his
initial visit, a urine culture grew Klebsiella
pneumoniae. He has a
2-year history of chronic prostatitis and has had four
episodes of
cystitis over the past year. His temperature is 36.9
C (98.4 F).
Examination, including prostate examination, shows no
abnormalities. Which of
the following is the most appropriate measure to
prevent recurrent
episodes of cystitis in this patient?
A
) Trimethoprim-sulfamethoxazole prophylaxis
B

) Insertion of a Foley catheter


C
) Total prostatectomy
D
) Transurethral prostatectomy
E
) No treatment is available
37. A 32-year-old woman comes for a routine health
maintenance
examination. Examination shows a bloody discharge
from the nipple of her
left breast. There is no palpable mass. On
questioning, she says that
she never noted the discharge and does not perform
breast
self-examinations. Which of the following is the most
likely diagnosis?
A
) Breast abscess
B
) Breast cyst
C
) Breast hematoma
D
) Fibroadenoma
E
) Fibrocystic changes of the breast
F
) Intraductal adenoma
38. A 35-year-old woman is brought to the physician
by her husband
because of increasing memory loss and involuntary
"dancing" movements
over the past 6 weeks. She has had difficulty
remembering things,
particularly recent events. She has had no
disturbances in sleep or appetite.
Her medical history is unremarkable. She does not
remember her mother
who died 25 years ago, but her father told her that
her mother had
similar symptoms several years before her death.
Neurologic examination
shows involuntary choreiform movements. She has a
childlike affect. She
describes her mood as good. On recall testing, she
can remember one out
of three objects after 3 minutes with distraction; she
cannot remember

the other objects with prompting. She cannot remember


the date of her
marriage or her previous address. The most likely
cause of these
symptoms is a lesion at which of the following
locations?
A
) Caudate nucleus
B
) Parietal lobe
C
) Prefrontal lobe
D
) Putamen
E
) Temporal lobe
39. A 67-year-old man comes to the physician
because of insomnia for
2 years. He goes to bed at 11:00 PM after taking a
bath but does not
fall asleep until midnight. He usually wakes up twice
each night: once
around 3:00 AM to void and again at 6:00 AM. He stays
in bed until his
alarm goes off at 7 AM. He is concerned because he
used to sleep 8
hours daily. He recently retired from his job as an
attorney. He takes
no medications. He drinks one beer with lunch 3 days
each week and one
glass of wine with dinner each evening. He walks 1
mile daily after
dinner. Physical examination shows no abnormalities.
His mood is
neutral, and cognition is intact. Urine toxicology
screening is negative.
Which of the following is the most likely cause of
this patient's
symptoms?
A
) Advanced sleep phase syndrome
B
) Alcohol abuse
C
) Delayed sleep phase syndrome
D
) Major depressive disorder
E
) Melatonin deficiency

F
) Poor sleep hygiene
G
) Normal aging
40. An asymptomatic 37-year-old woman comes for a
follow-up
examination 1 year after receiving a renal transplant.
Current medications
include felodipine, enalapril, cyclosporine,
pravastatin, and penicillin
G. She appears well. Her temperature is 36.8 C (98.2
F), blood
pressure is 160/95 mm Hg, pulse is 80/min, and
respirations are 12/min.
Examination shows no other abnormalities. Which of
the following
medications is the most likely cause of this patient's
hypertension?
A
) Cyclosporine
B
) Enalapril
C
) Felodipine
D
) Penicillin G
E
) Pravastatin
41. A 4080-g (9-lb) male newborn is delivered at
term to a
32-year-old woman, gravida 2, para 1. Apgar scores
are 8 and 9 at 1 and 5
minutes, respectively. Examination in the delivery
room shows fracture of
the right clavicle. Which of the following is the
most likely sequela
of this condition?
A
) Correction only with casting
B
) Correction only with physical therapy
C
) Left-hand dominance
D
) Permanent nerve damage
E
) Spontaneous healing without treatment

42. A 22-year-old man with schizoaffective disorder


is brought to
the emergency department 2 hours after the sudden
onset of neck and back
pain. The symptoms began after taking one of his
medications for his
psychiatric disorder. The dosage was increased
yesterday, but he does
not recall the names of either of his medications. He
has no other
history of serious illness. Physical examination
shows rigid contraction
of the neck and back muscles with arching. On mental
status
examination, he is alert and cooperative and hears a
faint voice that tells him to
wash windows. He is oriented to person, place, and
time. Which of the
following medications is most likely responsible for
this patient's
symptoms?
A
) Clozapine
B
) Haloperidol
C
) Lithium carbonate
D
) Trazodone
E
) Valproic acid
43. A 52-year-old woman, gravida 3, para 3, comes
to the physician
because of irregular vaginal bleeding over the past 2
months. She has
hypertension treated with enalapril and type 2
diabetes mellitus well
controlled with diet. Menopause occurred 2 years ago.
Her maternal aunt
had breast cancer at the age of 70 years. The patient
weighs 88 kg
(195 lb) and is 160 cm (63 in) tall. Examination
shows no other
abnormalities. An endometrial biopsy specimen shows
adenocarcinoma. Which of
the following is the most significant predisposing
factor for this
patient's endometrial cancer?
A
) Heredity
B
) Hypertension

C
) Parity
D
) Type 2 diabetes mellitus
E
) Weight
44. A 67-year-old man is brought to the emergency
department 2 hours
after the onset of weakness and double vision. He has
hypertension and
hyperlipidemia treated with metoprolol, captopril, and
atorvastatin.
His blood pressure is 190/106 mm Hg. Neurologic
examination shows
left-sided facial weakness including the forehead.
There is palsy of left
conjugate gaze, and the left eye fails to adduct on
right gaze.
Vertical eye movements are intact. Muscle strength is
3/5 in the right upper
and lower extremities. Deep tendon reflexes are
brisk, and Babinski's
sign is present on the right. Which of the following
is the most
likely location of this patient's lesion?
A
) Bilateral thalamic
B
) Left frontal
C
) Left pontine
D
) Right caudate
E
) Right midbrain
45. Six weeks after spontaneous drainage of an anal
abscess, a
32-year-old man has persistent blood-stained purulent
fluid on his
underwear. He has not had significant anal pain since
drainage of the boil.
Bowel movements are normal. Which of the following is
the most likely
diagnosis?
A
) Anal fissure
B
) Fistula in ano

C
) Pruritus ani
D
) Thrombosed external hemorrhoids
E
) Thrombosed internal hemorrhoids
46. A previously healthy 14-year-old girl is
brought to the
physician because of a 2-day history of fever and pain
and swelling of the
right knee. She remembers injuring the knee while
playing soccer last
week, but she was able to finish the game. She has no
history of rash or
joint pain. Her sister has inflammatory bowel
disease. The patient's
temperature is 39 C (102.2 F), blood pressure is
110/80 mm Hg, pulse is
95/min, and respirations are 20/min. Examination of
the right knee
shows swelling, tenderness, warmth, and erythema;
range of motion is
limited. Which of the following is the most
appropriate next step in
management?
A
) X-ray film of the right knee
B
) Gastrointestinal series with small-bowel
follow-through
C
) Nuclear scan of the right knee
D
) MRI of the right knee
E
) Antibiotic therapy
F
) Arthrocentesis

block 2
dolly123 - 11/07/06 17:58 #548028

1.

A 25-year-old man is brought to the emergency


department after being
discovered semiconscious and incoherent at home. On
arrival, he is
stuporous. His blood pressure is 105/70 mm Hg, pulse
is 80/min, and
respirations are 12/min. Examination shows cool, damp
skin. The pupils are
pinpoint and react sluggishly to light. Which of the
following is the
most likely substance taken?
A
) Alcohol
B
) Barbiturates
C
) Cocaine
D
) Heroin
E
) LSD

2. A 15-year-old girl is brought to the emergency


department by her
20-year-old sister because of a 1-week history of
fatigue, nausea, and
abdominal pain. Menarche was at the age of 12 years,
and her last
menstrual period was 3 weeks ago. She has not had
vaginal discharge. She
is sexually active, and she and her partner use
condoms inconsistently.
Her temperature is 37 C (98.6 F), blood pressure is
110/60 mm Hg, and
pulse is 95/min. Abdominal examination shows mild
bilateral lower
quadrant tenderness. Before a pelvic examination and
a pregnancy test can
be performed in this patient, consent must be obtained
from which of the
following?
A
) The court
B
) The patient
C
) The patient's parent
D
) The patient's sister

E
) No consent is necessary

3. A previously healthy 57-year-old man comes to


the physician
because of a nonpruritic rash over both legs for 1
week and a low-grade
fever for 2 days. He recently returned from a 2-week
canoe trip on a river
in Minnesota. His temperature is 37.2 C (99 F).
Examination shows a 4
x 6-cm, macular, dark pink, ovoid lesion on the right
posterior hip
with central clearing and a punctate eschar near the
center. There are
macular ring lesions with central clearing over the
medial tibia and
anterior thighs of the lower extremities. He has no
lymphadenopathy.
Which of the following is the most likely causal
organism?
A
) Borrelia burgdorferi
B
) Brucella melitensis
C
) Francisella tularensis
D
) Leptospira interrogans
E
) Rickettsia rickettsii
4. A 57-year-old woman comes to the physician
because of a 2-year
history of increasing menstrual flow. She has not had
hot flashes,
insomnia, or change in bowel or bladder function. Her
last menstrual period
was 2 weeks ago. Pelvic examination shows a
normal-appearing vulva,
vagina, and cervix. The uterus is consistent in size
with an 8-week
gestation. Bimanual examination shows a 4-cm, firm,
nontender left ovary. An
endometrial biopsy specimen shows atypical complex
endometrial
hyperplasia. Which of the following is the most likely
cause of this patient's
hyperplasia?
A
) Adrenal adenoma
B

) Brenner tumor
C
) Carcinoid tumor
D
) Granulosa cell tumor
E
) Hyperthecosis
F
) Islet cell tumor
G
) Sertoli-Leydig cell tumor
5. A 67-year-old woman comes to the physician
because of vaginal
bleeding for 10 days. She has been soaking one
sanitary pad daily.
Menopause was 10 years ago. Her last Pap smear 5
years ago showed normal
findings. Her temperature is 37 C (98.6 F), blood
pressure is 128/78 mm
Hg, pulse is 70/min, and respirations are 12/min.
Pelvic examination
shows a normal-appearing cervix and a small amount of
blood at the
cervical os. There is moderate thinning of the
vaginal mucosa. The uterus
is irregular and consistent in size with a 10-week
gestation. There
are no adnexal masses. Rectovaginal examination shows
no abnormalities.
Test of the stool for occult blood is negative. Which
of the following
is the most appropriate next step in diagnosis?
A
) Transvaginal ultrasonography of the pelvis
B
) Colposcopic-guided biopsy
C
) Endometrial biopsy
D
) Endometrial ablation
E
) Culdocentesis
6. During a routine examination, a 32-year-old man
has a blood
pressure of 120/80 mm Hg. He is concerned because his
father, grandfather,
and two uncles have hypertension. He works as a
systems programmer for

a large computer company and frequently has to meet


tight deadlines.
He has smoked one pack of cigarettes daily for 10
years. He is 4.5 kg
(10 lb) overweight and drinks three cups of coffee
daily. Which of the
following measures is most likely to reduce this
patient's risk for
hypertension over the next 5 years?
A
) Increase intake of dietary fiber
B
) Restrict caffeine
C
) Stress management
D
) Weight loss
7. A previously healthy 16-year-old boy comes to
the physician
because of persistent pain in his left testicle for 24
hours. He has not
had any penile discharge. There is no history of
trauma, but he plays
soccer every day. Two months ago, he had sexual
intercourse for the
first time, and he used a condom. Examination shows
an edematous,
erythematous, exquisitely tender left scrotum that is
lower than the right.
Elevating the left testicle relieves the pain. The
cremasteric reflex is
present. Urinalysis shows 10 leukocytes/hpf and 1+
leukocyte esterase.
A technetium 99m scan shows increased uptake in the
left testicle.
Which of the following is the most likely cause of the
pain?
A
) Cystitis
B
) Epididymitis
C
) Spermatocele
D
) Testicular torsion
E
) Testicular tumor
8. A 47-year-old woman comes to the physician
because of persistent

nonproductive cough for 6 weeks. She has not had


fever or weight loss.
She has hypertension treated with enalapril for the
past 3 months. She
does not smoke. There is no history of lung disease.
She weighs 54 kg
(120 lb) and is 163 cm (64 in) tall. Her temperature
is 37 C (98.6 F),
blood pressure is 130/80 mm Hg, pulse is 70/min, and
respirations are
12/min. Examination and an x-ray film of the chest
show no
abnormalities. Which of the following is the most
likely mechanism of this
patient's cough?
A
) Decreased plasma renin activity
B
) Decreased serum angiotensin II levels
C
) Increased serum angiotensin I levels
D
) Increased serum bradykinin levels
E
) Increased serum histamine levels
9. A 57-year-old man has been hospitalized for 2
days for treatment
of unstable angina pectoris. He is currently
receiving intravenous
heparin and undergoing evaluation for coronary artery
bypass grafting.
His blood pressure is 160/90 mm Hg, pulse is 88/min,
and respirations are
16/min. Laboratory studies show:
Platelet count 90,000/mm3
Prothrombin time 12 sec (INR=1.1)
Partial thromboplastin time 35 sec
Which of the following is the most likely cause of
these findings?
A
) Excessive platelet destruction
B
) Factor VIII deficiency
C
) Inadequate platelet production
D
) Uncontrolled activation of coagulation and
fibrinolytic cascades

E
) Vitamin K deficiency
10. A study is conducted to assess the
effectiveness of a new blood
test for early detection of prostate cancer. Ten
thousand healthy men
over the age of 50 years are randomly assigned to
receive either annual
rectal examination or annual screening with the new
blood test. After
5 years, results show that of the 50 men in the blood
test group that
were diagnosed with prostate cancer, 40 were living 2
years after the
diagnosis was made. In comparison, only 15 out of 45
men in the rectal
examination group survived 2 years after being
diagnosed with prostate
cancer. Researchers conclude that the blood test
increases survival
compared with rectal examination. Which of the
following potential flaws
is most likely to invalidate this conclusion?
A
) Age of the patients
B
) Diagnostic bias
C
) Lead time bias
D
) Recall bias
E
) Type II error
11.
A 62-year-old man comes to the emergency department
because of
progressive shortness of breath for 3 days. He has
not had chest pain,
orthopnea, or paroxysmal nocturnal dyspnea. He
completed chemotherapy for
small cell carcinoma of the lung 10 months ago. He
has a history of twice
nightly nocturia that has resolved over the past 3
days. He smoked two
packs of cigarettes daily for 30 years but quit 1 year
ago. His blood
pressure is 96/60 mm Hg, and pulse is 116/min. There
is jugular venous
distention to the angle of the jaw. The lungs are
clear to
auscultation. Cardiac examination shows distant heart
sounds, an S1 and S2, and no

gallops or rubs. The liver has a span of 12 cm and is


tender. There
is no pedal edema. Laboratory studies show:
Hemoglobin 10 g/dL
Serum
Na+ 135 mEq/L
Cl 110 mEq/L
K+ 4.2 mEq/L
HCO3 22 mEq/L
Urea nitrogen (BUN) 40 mg/dL
Creatinine 1.6 mg/dL
An ECG shows diminished amplitude of the QRS
complexes. An x-ray film
of the chest shows clear lung fields with an enlarged
cardiac
silhouette. Which of the following findings is most
likely to be accentuated?
A
) Cardiac output
B
) Fall in systolic arterial pressure with
inspiration
C
) Left ventricular end-diastolic pressure
D
) Mitral regurgitation
E
) Ventricular septal wall motion

12. A 35-year-old woman comes to the physician


because of two
12-hour episodes of dizziness over the past 3 months.
During episodes, she
experiences the acute onset of rotatory vertigo and
imbalance, decreased
hearing, tinnitus, a sense of fullness of the right
ear, and vomiting.
Examination shows a mild hearing loss of the right
ear. Which of the
following is the most likely diagnosis?
A
) Acoustic neuroma
B
) Benign positional vertigo
C
) Brain stem transient ischemic attacks
D
) Meniere's disease

E
) Viral labyrinthitis
3. An obese 33-year-old woman has had four 12-hour
episodes of
severe, sharp, penetrating pain in the right upper
quadrant of the abdomen
associated with vomiting but no fever. She has no
diarrhea, dysuria, or
jaundice and is asymptomatic between episodes. There
is slight
tenderness to deep palpation in the right upper
quadrant. Which of the
following is the most appropriate next step in
diagnosis?
A
) Supine and erect x-ray films of the abdomen
B
) Upper gastrointestinal series
C
) Ultrasonography of the upper abdomen
D
) CT scan of the abdomen
E
) HIDA scan of the biliary tract
14. An otherwise healthy 19-year-old woman comes to
the physician
because of a 3-year history of intermittent facial
blemishes. She drinks
wine occasionally on weekends. She takes no
medications. Examination
shows multiple 1- to 2-mm red and white papules and
larger red nodules
on the forehead and cheeks. Which of the following is
the most
appropriate initial pharmacotherapy?
A
) Oral isotretinoin
B
) Systemic corticosteroids
C
) Topical benzoyl peroxide
D
) Topical corticosteroids
E
) Topical metronidazole

15. A previously healthy 67-year-old man comes to


the physician
because of a 4-month history of hand weakness,
intermittent tingling of the
small fingers of his hands, and mild neck pain.
Examination shows
wasting, weakness, and fasciculations of the
interossei muscles. Sensation
is decreased to pinprick and vibration in the small
fingers of each
hand. Triceps tendon reflexes are decreased. Which
of the following is
the most likely diagnosis?
A
) Amyotrophic lateral sclerosis
B
) Cervical spondylosis
C
) Multiple sclerosis
D
) Myasthenia gravis
E
) Myasthenic (Lambert-Eaton) syndrome
F
) Myotonic muscular dystrophy
G
) Polymyalgia rheumatica
H
) Polymyositis
I
) Progressive neuropathic (peroneal) muscular
atrophY

16. A 19-year-old college student comes to student


health services
because of constant worrying since starting his
freshman year 6 months
ago. He attends college several hundred miles away
from his hometown.
He reports feeling constantly scrutinized by other
students and
professors. He feels embarrassed and anxious in class
and is in constant fear
of blushing. He has started to skip his classes
because of his anxiety
and worries that he will not be able to complete the
school year. He
drinks alcohol occasionally because it helps him
overcome his fear of
being embarrassed in front of others. He does not use

illicit drugs.
Physical examination shows no abnormalities. On
mental status
examination, he is mildly anxious. He exhibits fair
eye contact and shifts
uncomfortably in his chair. Laboratory studies are
within normal limits.
Which of the following is the most appropriate
pharmacotherapy for this
patient?
A
) Disulfiram
B
) Donepezil
C
) Fluoxetine
D
) Haloperidol
E
) Methylphenidate
F
) Valproic acid
17.
A 42-year-old woman comes for a follow-up examination.
Two weeks ago,
her blood pressure was 152/94 mm Hg during a routine
visit. Her blood
pressure today is 150/94 mm Hg, pulse is 76/min, and
respirations are
14/min. Examination shows no other abnormalities.
Serum studies show:
Na+ 142 mEq/L
Cl 105 mEq/L
K+ 4 mEq/L
HCO3 26 mEq/L
Urea nitrogen (BUN) 12 mg/dL
Glucose 101 mg/dL
Creatinine 0.8 mg/dL
An ECG shows no abnormalities. Which of the following
is the most
appropriate next step in management?
A
) Measurement of plasma renin activity
B
) Serum lipid studies
C
) 24-Hour urine collection for measurement of

metanephrine level
D
) Echocardiography
E
) Captopril renal scan
18. A 6-year-old boy with cystic fibrosis is
brought to the
physician by his mother because his skin has been cool
and clammy for 30
minutes. Earlier in the day, he had been playing
outdoors, and the
temperature was 99 F. When returning indoors, he was
thirsty and restless. His
blood pressure is 70/40 mm Hg, and pulse is 120/min.
Examination shows
dry mucous membranes. Serum sodium level is 128
mEq/L, and serum
chloride level is 87 mEq/L. Which of the following is
the most likely
explanation for these findings?
A
) Excessive sweat electrolyte level
B
) Excessive sweat volume
C
) Excessive urinary output
D
) Excessive vasopressor secretion
E
) Inadequate sweat production

19.
A 27-year-old primigravid woman at 29 weeks' gestation
comes to the
emergency department because of a 24-hour history of
increasingly severe
right-sided abdominal pain and no appetite. She has
vomited twice over
the past 4 hours. She has not had vaginal bleeding.
Her temperature
is 38.2 C (100.8 F). Examination shows exquisite
tenderness of the
right lateral flank and the fundus. There are no
peritoneal signs. Bowel
sounds are absent. The fetal heart rate is 144/min.
Laboratory
studies show:
Hematocrit 37%

Leukocyte count 16,000/mm3


Serum
Total bilirubin 1.1 mg/dL
Amylase 32 U/L
Lactate dehydrogenase 110 U/L
Urine WBC 35/hpf
Which of the following is the most likely diagnosis?
A
) Abruptio placentae
B
) Appendicitis
C
) Cholelithiasis
D
) Colitis
E
) Pyelonephritis
20. A 32-year-old woman has had a lump in her neck
for 6 months.
She has a 1.2-cm solitary left thyroid nodule.
Fine-needle aspiration
cytology of the mass is consistent with a low-grade
papillary malignancy.
Which of the following is the most appropriate next
step in management?
A
) Thyroid scan
B
) 131I therapy
C
) Propylthiouracil therapy
D
) Thyroxine therapy
E
) Left thyroid lobectomy
For each patient with vaginal bleeding, select the
most likely
diagnosis.
A
) Bacterial vaginosis
B
) Precocious puberty
C

) Urinary tract infection


D
) Vaginal foreign body
E
) Vaginal laceration
F
) von Willebrand's disease
21. A 12-year-old girl is brought to the physician
by her mother
because of heavy vaginal bleeding since her first
menstrual period began 1
week ago. She has to change sanitary pads every 2
hours, and her
mother is concerned that this is not normal. Her
blood pressure is 80/60 mm
Hg, and pulse is 110/min. Breast and axillary and
pubic hair
development are Tanner stage 4. Pelvic examination
shows normal external
genitalia, a small normal-appearing cervix, and a
small uterus. Her
hemoglobin level is 7 g/dL.

22. A 7-year-old girl is brought to the physician


by her mother 4
hours after she noticed that her daughter had blood on
her underwear.
She has no signs of sexual development. Examination
shows normal
external genitalia. There is a foul-smelling
discharge.
23.
A 10-year-old girl is brought to the physician because
of temperatures
to 40 C (104 F) and headaches for 1 week. Her mother
says that fever
and chills occur every other day and typically last
for several hours.
She appears ill and is lethargic during febrile
episodes. She and her
family returned from a trip to West Africa 2 weeks
ago. She currently
appears ill. Her temperature is 40 C (104 F), blood
pressure is 94/64
mm Hg, pulse is 146/min, and respirations are 20/min.
Examination shows
mild scleral icterus and conjunctival pallor. The
liver edge is
palpated 3 cm below the right costal margin, and the
spleen tip is palpated 3
cm below the left costal margin. Laboratory studies
show:

Hemoglobin 8 g/dL
Leukocyte count 6400/mm3
Segmented neutrophils 46%
Eosinophils 5%
Lymphocytes 40%
Monocytes 9%
Serum
Na+ 132 mEq/L
Cl 98 mEq/L
K+ 4.2 mEq/L
HCO3 16 mEq/L
Urea nitrogen (BUN) 21 mg/dL
Bilirubin
Total 5.2 mg/dL
Direct 0.8 mg/dL
Aspartate aminotransferase (AST, GOT) 64 U/L
Alanine aminotransferase (ALT, GPT) 98 U/L
Urine
Color tea
Blood strongly positive
RBC occasional
Which of the following measures is most likely to have
prevented this
condition?
A
) Hepatitis A vaccine
B
) Typhoid vaccine
C
) Oral isoniazid prophylaxis
D
) Oral mefloquine prophylaxis
E
) Oral trimethoprim-sulfamethoxazole prophylaxis
F
) Intramuscular immune globulin

24. A 6-month-old boy is brought to the physician


because of
seizures over the past week. His parents note that
during this period, he has
had 10 to 20 episodes of throwing out his arms for 1
to 2 seconds then
crying as if he were afraid of something. He was born
at term
following an uncomplicated pregnancy and delivery.
Development is appropriate
for age. Two days ago, treatment with amoxicillin was
begun for otitis
media. He appears pale and apathetic. Examination
shows multiple
white patches over the skin that become more distinct

with Wood's lamp


examination. Neurologic examination shows no
abnormalities. An MRI of the
brain is shown. Which of the following is the most
likely diagnosis?
A
) Congenital cytomegalovirus encephalopathy
B
) Congenital toxoplasmosis encephalopathy
C
) Hypoxic-ischemic encephalopathy
D
) Neurofibromatosis
E
) Tuberous sclerosis
25.
An ELISA is used to detect Chlamydia trachomatis
infection in patients
seen at a family planning clinic. In the first 500
patients, cultures
are also done to check the accuracy of the ELISA. The
results are
listed below:
Chlamydia culture
+
ELISA
+
38
5
2
455
Which of the following is the positive predictive
value of the ELISA?
A
) 2/40
B
) 5/43
C
) 38/40

D
) 38/43
E
) 38/493

26. A 27-year-old woman is brought to the emergency


department by
her mother who found her comatose 30 minutes ago. Her
mother says that
her daughter had been having lower abdominal pain and
vaginal bleeding
over the past week. The patient had an ectopic
pregnancy 2 years ago
and was also treated with doxycycline for pelvic
inflammatory disease at
that time. Her blood pressure is 40/20 mm Hg, pulse
is 160/min, and
respirations are 24/min. The abdomen is distended and
rigid with
decreased bowel sounds. Hemoglobin level is 4.2 g/dL,
and leukocyte count is
12,500/mm3. Culdocentesis is positive. Which of the
following is the
most appropriate next step in management?
A
) Bromocriptine therapy
B
) Clomiphene therapy
C
) Conjugated estrogen therapy
D
) Ergot derivative therapy
E
) Hysteroscopy
F
) Laparoscopy
G
) Dilatation and curettage
H
) Endometrial ablation
I
) Exploratory laparotomy
J
) Total abdominal hysterectomy

27. A 43-year-old man comes to the emergency

department because of
fever, chills, malaise, cough, and pleuritic chest
pain for 3 days; the
cough is productive of foul-smelling, purulent sputum.
He has
alcoholism. His temperature is 39.2 C (102.6 F),
blood pressure is 110/70 mm
Hg, pulse is 120/min, and respirations are 14/min.
Hemoglobin level is
15 g/dL, and leukocyte count is 25,000/mm3. An ECG
shows normal
findings. An x-ray film of the chest shows a 4-cm
cavity in the superior
segment of the right lower lobe and an air-fluid
level. While awaiting
results of sputum culture, which of the following is
the most appropriate
next step in management?
A
) Observation
B
) Broad-spectrum antibiotic therapy
C
) Isoniazid therapy
D
) Intravenous amphotericin B therapy
E
) Tube thoracostomy
28.
A 12-year-old girl with type 1 diabetes mellitus is
brought to the
physician because of shortness of breath and fatigue
for 1 day. Since
menarche began 4 months ago, she has had one episode
of diabetic
ketoacidosis per month; prior to that she had been
stable. Her blood pressure is
110/70 mm Hg, pulse is 140/min, and respirations are
36/min. She
appears to be moderately dehydrated. Laboratory
studies show:
Serum
Na+ 132 mEq/L
Cl 90 mEq/L
K+ 5.9 mEq/L
HCO3 6 mEq/L
Urea nitrogen (BUN) 48 mg/dL
Glucose 600 mg/dL
Creatinine 2.8 mg/dL
Urine
Glucose 4+
Ketones 3+

Protein 1+
Which of the following is the most likely underlying
cause of this
patient's respiratory disorder?
A
) Acute interstitial nephritis
B
) Acute renal failure
C
) Diabetic nephropathy
D
) Hyperinsulinemia
E
) Increased plasma ketone level
29. A 28-year-old woman develops fatigue and
orthopnea 3 weeks after
the uncomplicated delivery of her first child. Her
blood pressure is
115/78 mm Hg, and pulse is 112/min. Crackles are
heard at both lung
bases. The point of maximal impulse is laterally
displaced and diffuse;
an S3 is present. There is 2+ pedal edema. An x-ray
film of the chest
shows vascular cephalization of pulmonary vasculature
and Kerley B lines.
Which of the following is the most appropriate next
step in management?
A
) Measurement of serum creatine kinase and lactate
dehydrogenase
activities
B
) Echocardiography
C
) Dipyridamole thallium scan
D
) Ventilation-perfusion lung scans
E
) Myocardial biopsy
30. A previously healthy 22-year-old woman comes to
the physician
because of a paroxysmal cough for 2 weeks. She works
as an oncology
nurse. Her temperature is 37 C (98.6 F). Examination
shows no
abnormalities except for intermittent coughing spells

during the examination. An


x-ray film of the chest is normal. Cultures of
nasopharyngeal
secretions grow Bordetella pertussis. Which of the
following is the most
appropriate pharmacotherapy for this patient?
A
) Amoxicillin
B
) Amoxicillin-clavulanate
C
) Cefprozil
D
) Erythromycin
E
) Penicillin G
31. A 5-year-old girl is brought to the physician
because of a 2-day
history of temperatures to 39.5 C (103.1 F) and pain
in the right side.
She has had two episodes of vomiting during this
period but no diarrhea
or symptoms of upper respiratory tract infection. She
has
vesicoureteral reflux and a history of recurrent
urinary tract infections. She
appears ill. Her temperature is 39.8 C (103.6 F),
blood pressure is
110/60 mm Hg, pulse is 150/min, and respirations are
25/min. Examination
shows right-sided costovertebral angle tenderness. In
addition to
obtaining urine cultures, which of the following is
the most appropriate next
step in management?
A
) Intravenous pyelography
B
) Renal ultrasonography
C
) Voiding cystourethrography
D
) Intramuscular antibiotic therapy and
reexamination in 24 hours
E
) Intravenous antibiotic therapy
32. A 10-year-old boy is brought to the physician
because of

weakness and decreased appetite for 3 months. He


weighs 30 kg (66 lb) and is
142 cm (56 in) tall; he has had a 2.3-kg (5-lb) weight
loss since his
last examination 6 months ago. He appears thin. Deep
tendon reflexes
are brisk. Chvostek's sign is positive. His serum
calcium level is 6.5
mg/dL, and serum intact parathyroid hormone level is
190 pg/mL
(N=965). In addition to calcium supplementation,
which of the following is
the most appropriate next step in management?
A
) Oral phosphate supplementation
B
) Oral vitamin D supplementation
C
) Intranasal calcitonin therapy
D
) Parenteral bisphosphate therapy
E
) Parenteral parathyroid hormone therapy
33. A healthy 8-year-old girl is brought to the
physician in July
for a well-child examination. Her mother says that
her daughter is
spending the summer at a nearby lake. Over the past
month, she has had two
episodes of painful sunburn despite her mother's
efforts, including SPF
25 sunblock just before she goes swimming and urging
her to wear a hat
and long-sleeved garments. The child takes no
medications. She has
blond hair, blue eyes, and a fair complexion. The
mother seeks advice
about preventing further sun damage to her child's
skin. Which of the
following is the most appropriate recommendation?
A
) Prohibit swimming on cloudless days
B
) Apply the sunblock lotion 45 minutes before
swimming
C
) Change to a higher-level SPF lotion
D
) Apply Burrow's solution compresses after each
overexposure

E
) Daily use of antioxidant vitamin supplement
F
) Early treatment of any sun overexposure with
topical
corticosteroids

34. Two days after admission to the hospital


because of a 3-day
history of slurred speech, double vision, and
dysphagia, a 24-year-old
woman becomes quadriplegic and requires intubation and
mechanical
ventilation. Her medical history is unremarkable.
One week ago, she attended a
family picnic; several of her family members have had
abdominal cramps
and diarrhea since the picnic. Her temperature is 37
C (98.6 F), blood
pressure is 120/80 mm Hg, and pulse is 120/min.
Examination shows dry
mucous membranes, large unreactive pupils,
ophthalmoplegia, and
profound facial weakness. There is areflexia,
quadriplegia, and no movement
of the palate and tongue. Sensation is normal.
Babinski's sign is
absent. Which of the following is the most
appropriate pharmacotherapy?
A
) Antitoxin
B
) Azathioprine
C
) Interferon
D
) Pyridostigmine
E
) Riluzole

35. A 2325-g (5 lb 2 oz) male newborn is delivered


at 33 weeks'
gestation; Apgar scores are 7 and 8 at 1 and 5
minutes, respectively. The
13-year-old mother had no prenatal care and did not
know how much
weight she gained. During the pregnancy, the mother
smoked marijuana and
took over-the-counter vitamins occasionally; she did
not drink alcohol

and had no illness except for an upper respiratory


tract infection 4
months ago. She did not know she was pregnant until 2
weeks ago; her
family is unaware of her condition. She has had one
sexual partner. During
the hospital stay, the newborn and his mother have no
complications.
The newborn is at greatest risk for morbidity and
mortality from which
of the following?
A
) Child abuse
B
) Congenital syphilis
C
) Hypocalcemia
D
) Lead poisoning
E
) Seizures
36. A 2-month-old girl is brought to the physician
because of a
2-week history of progressive difficulty breathing and
poor feeding. She
has had rapid and labored breathing and sweating
during feedings. There
is no history of fever or viral illness. Her
temperature is 37 C (98.6
F), blood pressure is 80/60 mm Hg, pulse is 130/min,
and respirations
are 40/min. Bilateral crackles are heard at both lung
bases. A grade
4/6 holosystolic murmur is heard along the left
sternal border; the
precordium is hyperdynamic. The liver edge is
palpated 4 cm below the
right costal margin. An x-ray film of the chest shows
cardiomegaly and
pulmonary congestion. Which of the following is the
most likely
underlying mechanism for this child's condition?
A
) Decreased systemic vascular resistance
B
) Increased pulmonary vascular resistance
C
) Increased systemic vascular resistance
D
) Intracardiac left-to-right shunt

E
) Intracardiac right-to-left shunt

37. A 37-year-old man comes to the physician 6


months prior to
traveling to sub-Saharan Africa for 1 year. He has no
history of hepatitis
and has no high-risk behavior for hepatitis B. Which
of the following
is the most appropriate recommendation to prevent
hepatitis during and
after his trip?
A
) Immune globulin
B
) Hepatitis B immune globulin (HBIG) only
C
) Hepatitis B vaccine series only
D
) Hepatitis B vaccine series and HBIG
E
) Hepatitis B vaccine series and hepatitis A
vaccine

38.
A 67-year-old man comes to the physician because of a
2-month history
of progressive shortness of breath. He has had a
4.5-kg (10-lb) weight
loss over the past 4 months. He has not had chest
pain. He has
congestive heart failure treated with furosemide,
digoxin, and enalapril. He
has smoked two packs of cigarettes daily for 30 years.
He appears
alert and is in no acute distress. His temperature is
37.2 C (99 F), blood
pressure is 140/85 mm Hg, pulse is 84/min, and
respirations are 18/min.
Examination shows no jugular venous distention. There
is dullness to
percussion, and breath sounds are decreased at the
left base. Cardiac
examination shows a laterally displaced point of
maximal impulse, normal
S1 and S2, and an S3 at the apex. There is 1+ edema
over the
extremities. An x-ray film of the chest shows an
enlarged cardiac silhouette,
left hilar fullness, and a moderate-sized left pleural
effusion.
Thoracentesis yields straw-colored fluid. Laboratory

studies show:
Serum
Glucose 90 mg/dL
Protein 7 g/dL
Lactate dehydrogenase 300 U/L
Pleural fluid
pH 7.25
Glucose 75 mg/dL
Protein 4.5 g/dL
Lactate dehydrogenase 280 U/L
Leukocyte count 2000/mm3
Segmented neutrophils 15%
Lymphocytes 85%
A Gram's stain and acid-fast stains are negative for
any organisms.
Which of the following is the most likely cause of
this patient's pleural
effusion?
A
) Bacterial pneumonia
B
) Collagen vascular disease
C
) Congestive heart failure
D
) Malignancy
E
) Pulmonary embolus with infarction
F
) Viral pleuritis
39. A previously healthy 67-year-old woman is
admitted to the
hospital because of a 2-week history of dark urine,
clay-colored stools, and
increasing jaundice. She has had a 9-kg (20-lb)
weight loss over the
past 2 months due to loss of appetite. She also has
had generalized
itching that is most severe at night. She has not had
any abdominal pain.
Examination shows no abnormalities except for
jaundice. Which of the
following is the most likely diagnosis?
A
) Common bile duct stone
B
) Drug-induced jaundice
C

) Hemolytic jaundice
D
) Pancreatic carcinoma
E
) Viral hepatitis
40. A 2-year-old boy is brought to the physician
because of fever
and listlessness for 12 hours. He has had recurrent
episodes of
pneumonia and otitis media over the past year. Two
maternal uncles died of
pneumonia in early childhood. One year ago, he was at
the 50th percentile
for height and weight; he is currently at the 25th
percentile for
height and 10th percentile for weight. He appears
ill. His temperature is
39 C (102.2 F), blood pressure is 60/40 mm Hg, pulse
is 160/min, and
respirations are 36/min. Examination shows cool and
mottled extremities.
A blood culture grows Streptococcus pneumoniae. Serum
IgE, IgG, and
IgM levels are markedly decreased. Which of the
following diagnostic
tests is most likely to be abnormal?
A
) Candidal skin test
B
) Flow cytometry identification of B lymphocytes
C
) Nitroblue tetrazolium test
D
) T-lymphocyte receptor stimulation by concanavalin
A
E
) Total serum hemolytic complement assay

41. A 1-week-old newborn has had poo


feeding,
vomiting, and
progressive lethargy over the past 4 days. She was
born at term; pregnancy,
labor, and delivery were uncomplicated, and she had no
congenital
anomalies. She is being breast-fed. She has a
healthy 2-year-old brother; a
sister died at 10 days of age after a full-term birth.
Examination

shows decreased muscle tone and poor responsiveness;


reflexes are normal.
Serum bicarbonate level is 8 mEq/L, pH is 7.15, and
plasma ammonia
level is 10 times the upper limit of normal. Which of
the following is the
most likely cause?
A
) Mitochondrial disorder
B
) Mucopolysaccharidoses disorder
C
) Organic acid metabolism disorder
D
) Renal tubular acidosis
E
) X-linked leukodystrophy
For each patient with an infection, select the most
likely causal
organism.
A
) Chlamydia trachomatis
B
) Clostridium botulinum
C
) Clostridium tetani
D
) Group A streptococcus
E
) Group B streptococcus
F
) Listeria monocytogenes
G
) Neisseria gonorrhoeae
H
) Streptococcus pneumoniae
42.
A premature 18-hour-old newborn is intubated and
mechanically
ventilated because of progressive respiratory
distress. Labor was complicated
by maternal fever and increased leukocyte count;
membranes ruptured 36
hours before delivery. His blood pressure and urine

output have
decreased since birth. The newborn appears acutely
ill, and peripheral pulses
are not palpable; the skin is pale, cool, and mottled.
The liver edge
is palpable 4 cm below the right costal margin. His
leukocyte count is
5000/mm3, platelet count is 48,000/mm3, and partial
thromboplastin time
is 60 sec. An x-ray film of the chest shows diffuse,
bilateral,
interstitial infiltrates.

43. A 6-year-old boy has been unable to walk for 2


days because of a
sore right knee. Three weeks ago he had a sore throat
and fever that
resolved within 2 days. He appears acutely ill. His
temperature is 39.2
C (102.5 F), and pulse is 120/min. A grade 2/6
pansystolic murmur is
heard at the apex. The right knee is red, tender, and
swollen; any
motion is painful. His leukocyte count is 15,000/mm3,
and erythrocyte
sedimentation rate is 120 mm

44. A 70-year-old woman comes to the physician


because of fatigue
and increasing difficulty in her daily functioning
over the past 2
months. During the day, she lies in bed for hours and
cries. She has had a
10.9-kg (24-lb) weight loss over the past 4 weeks,
eats only cookies,
has lost interest in almost everything, and wishes to
kill herself. Her
husband died 4 months ago. Physical examination and
laboratory studies
show normal findings except for decreased serum
albumin and total
protein levels. Which of the following is the most
likely diagnosis?
A
) Adjustment disorder with depressed mood
B
) Bereavement
C
) Bipolar disorder, depressed
D
) Dysthymic disorder

E
) Major depressive disorder

45. A 42-year-old woman comes to the physician


because of increasing
low back pain for 2 days. She is a daily intravenous
drug user. She
has a history of pyelonephritis, abscesses at
injection sites, and
pelvic inflammatory disease. Her temperature is 39 C
(102.2 F), blood
pressure is 130/70 mm Hg, pulse is 84/min, and
respirations are 20/min.
Examination shows warm, dry skin, a supple neck, and
no jugular venous
distention. There is tenderness over L4. Pelvic
examination shows mild
erythema around the cervical os and scant discharge;
there is no adnexal
or cervical motion tenderness. Her hematocrit is 30%,
leukocyte count
is 10,600/mm3, and serum glucose level is 110 mg/dL.
Urinalysis is
within normal limits. Which of the following is the
most appropriate next
step in management?
A
) Echocardiography
B
) Renal ultrasonography
C
) MRI of the back
D
) Colposcopy
E
) Laparoscopy
46. A 13-year-old boy is brought to the physician
by his mother
because of frequent headaches over the past 5 weeks.
He describes the
headaches as dull pain across his forehead; they occur
four to five times
weekly. Acetaminophen does not relieve the pain. His
mother states
that occasionally he has nausea without vomiting. He
has schizophrenia
well controlled with risperidone. Physical
examination shows no
abnormalities. An MRI of the brain is most likely to
show which of the
following?

A
) Bilateral increased caudate nuclei
B
) Decreased cerebellar volume
C
) Hippocampal symmetry
D
) Increased lateral ventricle size
E
) Multiple white-matter hyperintensities
Report Abuse
* block 3:--dolly123 - 11/07/06 17:59 #548031

1. A 42-year-old woman, gravida 2, para 2, comes to


the physician
because of a 3-month history of swelling of her legs
and mild abdominal
pain and bloating. Abdominal examination shows no
abnormalities.
Rectovaginal examination shows fullness in the right
adnexa. Transvaginal
ultrasonography shows an irregular mass in the right
ovary with some
solid components to a predominantly cystic lesion.
Her serum CA 125 level
is 120 U/mL (N<35). Treatment with which of the
following is most
likely to have prevented this patient's symptoms?
A
) Antiestrogens
B
) Antiprogestationals
C
) Medroxyprogesterone
D
) Oral contraceptives
E
) Ovulation-inducing drugs

2. A 15-year-old girl is brought to the physician 3


months after she
had a blood pressure of 150/95 mm Hg at a routine
examination prior to
participation in school sports. She is asymptomatic
and has no history

of serious illness. Twelve months ago, she was


diagnosed with a
urinary tract infection and treated with oral
trimethoprim-sulfamethoxazole.
She currently takes no medications. Subsequent blood
pressure
measurements on three separate occasions since the
last visit have been: 155/94
mm Hg, 145/90 mm Hg, and 150/92 mm Hg. She is at the
50th percentile
for height and 95th percentile for weight. Her blood
pressure today is
150/90 mm Hg confirmed by a second measurement, pulse
is 80/min, and
respirations are 12/min. Examination shows no other
abnormalities. Her
hematocrit is 40%. Urinalysis is within normal limits.
Cardiac and
renal ultrasonography shows no abnormalities. Which
of the following is
the most appropriate next step in management?
A
) Exercise and weight reduction program
B
) Measurement of urine catecholamine levels
C
) Measurement of urine corticosteroid levels
D
) Captopril therapy
E
) Hydrochlorothiazide therapy

3. Over the past 4 years, a 40-year-old woman has


had increasing
episodes of loss of urine and difficulty emptying her
bladder. She has
had no dysuria. She has a 30-year history of type 1
diabetes mellitus.
She weighs 66 kg (145 lb) and is 175 cm (69 in) tall.
Pelvic
examination shows a moderate cystocele. Postvoiding
catheterization yields 700
mL of clear urine. Which of the following is the most
likely cause of
the patient's genitourinary symptoms?
A
) Carcinoma of the bladder
B
) Detrusor instability
C
) Neurogenic bladder

D
) Urethral diverticulum
E
) Uterine prolapse

4. A 57-year-old man comes to the emergency


department because of an
episode of confusion, nervousness, sweating, and
palpitations 1 hour
ago. He has had four similar episodes over the past 3
weeks; they last 2
to 3 minutes and are relieved by ingesting solid food
or liquids. His
blood pressure is 140/74 mm Hg, pulse is 76/min, and
respirations are
18/min. Examination shows no other abnormalities.
The most appropriate
next step in diagnosis is serum measurement of which
of the following?
A
) Fasting gastrin level
B
) Fasting insulin and glucose levels
C
) Glucagon level
D
) Glucose and somatostatin level
E
) Glucose and vasoactive intestinal
5. An otherwise healthy 15-year-old girl is brought
to the physician
because she has never had a menstrual period. She
reports that breast
development started 1 year ago and pubic and axillary
hair development
began 6 months ago. Examination shows normal
genitalia. Breast
development is Tanner stage 4, and pubic hair
development is Tanner stage 3.
Which of the following is the most appropriate next
step in management?
A
) Reexamination in 1 year if the patient has not
had menarche
B
) Measurement of serum follicle-stimulating hormone
and luteinizing
hormone levels
C
) Measurement of serum thyroid-stimulating hormone

and prolactin
levels
D
) Karyotype analysis
E
) Progesterone withdrawal test
F
) Pelvic ultrasonographypolypeptide levels

6.
A 2-month-old boy is brought to the physician because
of a 6-week
history of persistent diarrhea and vomiting, most
pronounced after formula
feedings. He has had a 113-g (4-oz) weight loss since
birth. He
currently weighs 3100 g (6 lb 13 oz) and is 51 cm (20
in) in length. He
appears irritable. Examination shows jaundice. The
lungs are clear to
auscultation. No murmurs are heard. The liver is
palpated 2 to 3 cm
below the right costal margin, and the spleen is
palpated 1 to 2 cm below
the left costal margin. Laboratory studies show:
Serum
Glucose 35 mg/dL
Bilirubin (total) 2.3 mg/dL
Urine
Glucose negative
Reducing substances 3+
Which of the following is the most likely mechanism of
these findings?
A
) Decreased gluconeogenesis
B
) Decreased insulin secretion
C
) Increased glucagon secretion
D
) Increased gluconeogenesis
E
) Increased insulin secretion
F
) Insulin resistance

7. A 75-year-old man has had hypertension for 25


years. There is an
unusually prominent pulsation of the abdominal aorta
in the upper
midabdomen. A systolic bruit is heard at this site.
Femoral, popliteal, and
pedal pulses are present. Which of the following is
the most
appropriate initial diagnostic study?
A
) X-ray film of the abdomen
B
) Abdominal ultrasonography
C
) Doppler ultrasonography of the arteries of the
legs
D
) Abdominal aortography
E
) Intravenous pyelography

8. A 14-year-old boy is brought to the physician


because of constant
right knee pain for 2 weeks. The pain is not relieved
by rest or
analgesics. There is no history of trauma. He is at
the 50th percentile
for height and above the 95th percentile for weight.
He walks with a
limp but is not in distress. His vital signs are
within normal limits.
Examination shows full range of motion of both knees;
passive abduction
and internal rotation of the right hip produce pain.
There is no
swelling of either knee or hip. Neurologic
examination shows no
abnormalities. Which of the following is the most
likely diagnosis?
A
) Juvenile rheumatoid arthritis
B
) Osgood-Schlatter disease
C
) Septic arthritis
D
) Slipped capital femoral epiphysis

E
) Toxic synovitis

9. A previously healthy 72-year-old man comes to


the physician
because of a 2-year history of hand tremors and
progressive difficulty
walking. He lives alone and has no close relatives.
He is alert and
oriented. Physical examination shows a decreased rate
of eye blinking.
Neurologic examination shows masked facies and a
pill-rolling resting
tremor of both hands. There is cogwheel rigidity of
the upper extremities
and generalized bradykinesia. His handwriting has
become small and
illegible. He has a slow, shuffling, festinating gait
with a tendency to
lean forward. Postural reflexes are impaired. This
patient is at
greatest risk for injury due to which of the
following?
A
) Bradykinesia
B
) Cogwheel rigidity
C
) Decreased rate of eye blinking
D
) Postural reflex impairment
E
) Tremor

10. A 47-year-old woman comes for a routine health


maintenance
examination. She has a 10-year history of type 2
diabetes mellitus that is
well controlled with an oral hypoglycemic agent. Her
mother died of a
myocardial infarction at the age of 38 years. The
patient weighs 82 kg
(180 lb) and is 163 cm (64 in) tall. Her blood
pressure is 150/95 mm
Hg. Examination shows multiple small skin tags below
the eyebrows and
on the nose and eyelids. Which of the following is
the most appropriate
next step in diagnosis?
A
) Measurement of serum follicle-stimulating hormone

level
B
) Serum lipid studies
C
) 24-Hour urine collection for measurement of
creatinine clearance
D
) X-ray film of the chest
E
) ECG
F
) No further studies indicated

11.
A previously healthy 15-year-old boy is brought to the
physician
because of a 5-day history of fever, intractable
nausea and vomiting, sore
throat, and muscle pain. His mother has been giving
him ibuprofen and
amoxicillin that was remaining from a previous
streptococcal throat
infection. He appears ill, and his lips are parched.
His temperature is
38.9 C (102 F), blood pressure is 120/74 mm Hg while
supine and 100/70 mm
Hg while standing, and pulse is 92/min while supine
and 120/min while
standing. Examination shows dry mucous membranes.
The oropharynx is
erythematous without exudate. There is shotty
cervical adenopathy. The
abdomen is soft without organomegaly. Laboratory
studies show:
Serum
Na+ 138 mEq/L
Cl 98 mEq/L
K+ 3.4 mEq/L
HCO3 21 mEq/L
Urea nitrogen (BUN) 55 mg/dL
Glucose 105 mg/dL
Creatinine 1.3 mg/dL
Amylase 40 U/L
Urine
Ketones moderate
WBC negative
RBC negative
Na+ 8 mEq/L
Protein negative
Which of the following is the most likely explanation

for this
patient's renal insufficiency?
A
) Acute tubular necrosis
B
) Amoxicillin-induced acute interstitial nephritis
C
) Ibuprofen-induced renal failure
D
) Post-streptococcal glomerulonephritis
E
) Severe volume depletion

12. A 77-year-old woman is brought to the emergency


department after
collapsing at home. Six hours ago, she had the sudden
onset of massive
bright red rectal bleeding. On arrival, her blood
pressure is 90/60 mm
Hg, and pulse is 120/min. Abdominal examination shows
no
abnormalities. Insertion of a nasogastric tube yields
clear aspirate. Her
hematocrit is 28%. Which of the following is the most
likely diagnosis?
A
) Colon cancer
B
) Diverticulosis
C
) Duodenal ulcer
D
) Hemorrhoids
E
) Inflammatory bowel disease

13. A previously healthy 27-year-old woman comes to


the emergency
department 1 hour after a 30-minute episode of
shortness of breath,
nausea, diaphoresis, and crushing substernal chest
pain radiating to the
left shoulder; the episode resolved spontaneously.
She is currently
asymptomatic. Her blood pressure is 110/84 mm Hg,
pulse is 70/min, and
respirations are 16/min. Examination shows no
abnormalities. An ECG shows

a normal sinus rhythm and no other abnormalities. Ten


minutes later,
she sits up in bed stating that her symptoms have
returned. Her blood
pressure now is 150/105 mm Hg, pulse is 120/min, and
respirations are
24/min. A repeat ECG shows 5-mm ST-segment elevation
in leads II, III,
and aVF. Her symptoms resolve after administration of
aspirin and
nitroglycerin. An angiography is ordered and shows no
evidence of coronary
atherosclerosis. Which of the following is the most
appropriate
pharmacotherapy?
A
) Angiotensin-converting enzyme (ACE) inhibitor
B
) Aspirin
C
) Benzodiazepine
D
) Calcium-channel blocking agent
E
) Corticosteroids
F
) Thiazide diuretic

Na+(mEq/L) Cl(mEq/L) K+(mEq/L) HCO3(mEq/L)


pH Specific
gravity
A
)
132 89 2.8 39 5.0 1.025
B
)
133 110 3.9 16 6.0 1.015
C
)
163 117 4.3 22 5.5 1.003
D
)
165 115 4.5 19 5.0 1.030
14. An 8-week-old boy is brought to the physician
by his mother
because of persistent vomiting for 2 days. He has
spit up intermittently
since birth. He has had no diarrhea. He appears

irritable. His
temperature is 38 C (100.4 F), blood pressure is 90/60
mm Hg, pulse is
130/min, and respirations are 25/min. Examination
shows a sunken anterior
fontanelle. Mucous membranes are dry. The lungs are
clear to
auscultation. Heart sounds are normal. The abdomen
is scaphoid, and bowel
sounds are active.

15. A 7-week-old boy is brought to the physician


because of fever,
irritability, and lethargy for 3 days. He has had no
vomiting,
diarrhea, or symptoms of an upper respiratory tract
infection. He always seems
hungry to his mother despite being fed 3 ounces of
formula every 3
hours. His mother also notes that he has many wet
diapers throughout the
day and night. He currently appears irritable. His
temperature is 37.8
C (100 F), blood pressure is 80/50 mm Hg, pulse is
150/min, and
respirations are 25/min. Examination shows a sunken
anterior fontanelle.
Mucous membranes are dry. There is tenting of the
skin. Examination
shows no other abnormalities. Urinalysis is negative
for glucose and
protein.

16.
A 57-year-old woman is extubated and transferred to
the recovery room
after a cholecystectomy. She appears restless. Her
blood pressure is
120/70 mm Hg, pulse is 80/min, and respirations are
10/min. Arterial
blood gas analysis on room air shows:
pH 7.24
PCO2 85 mm Hg
PO2 60 mm Hg
Intravenous naloxone therapy is begun, but she does
not improve. Which
of the following is the most appropriate next step in
management?
A
) Encouraging deep breathing and cough
B
) Administration of 40% oxygen via nasal cannula

C
) Administration of furosemide
D
) Transfusion of 1 unit of packed red blood cells
E
) Reintubation and mechanical ventilation

17. A previously healthy 24-year-old woman is


brought to the
physician by her husband because of several episodes
of loss of consciousness
over the past 4 days. Her husband reports that during
episodes, she
jerks her arms and legs wildly. Each episode lasts up
to 1 hour; between
episodes, her behavior is normal. She is planning to
move to another
state because of her husband's work. She has been
extremely anxious and
upset about the move because she will have to leave
her mother, who was
recently diagnosed with breast cancer. There is no
family history of
seizure disorder. Her temperature is 36.7 C (98 F),
blood pressure is
130/80 mm Hg, pulse is 84/min, and respirations are
18/min. Neurologic
examination shows no abnormalities.
Electroencephalography shows
normal findings during an episode of shaking. Which
of the following is the
most likely underlying cause?
A
) Catatonia
B
) Complex partial seizure
C
) Conversion reaction
D
) Dissociative fugue
E
) Malingering
F
) Tonic-clonic seizure

18. A 37-year-old man comes to the physician


because of a 3-month
history of increasing pain of his shoulders and upper
arms. Over the

past 2 weeks, he has also had difficulty lifting heavy


objects off shelves
in his garage. Two days ago, he burned his hand by
touching a hot pan
and felt little discomfort. He sustained a concussion
in a motor
vehicle collision at the age of 29 years. He has no
allergies. He takes no
medications. He does not smoke cigarettes or use
illicit drugs, and he
drinks one to two beers daily. Muscle strength is
decreased equally in
both arms. Temperature and pain sensation are
decreased in both hands,
but light touch is normal. Muscle strength and
sensation are normal in
the lower extremities. Neurologic examination shows
no other
abnormalities. Which of the following is the most
likely diagnosis?
A
) Alcoholic peripheral neuropathy
B
) Ankylosing spondylitis
C
) Guillain-Barr syndrome
D
) Herniated intervertebral disc
E
) Multiple sclerosis
F
) Polymyositis
G
) Syringomyelia

19. A 62-year-old woman with ovarian cancer comes


to the emergency
department because of fever for 2 days. Ten days ago,
she received
chemotherapy with paclitaxel and carboplatin. She
feels tired but has not
had nausea or vomiting. Her temperature is 39.5 C
(103.1 F), blood
pressure is 100/60 mm Hg, and pulse is 115/min. The
lungs are clear to
auscultation. Examination shows a soft, nontender
abdomen. Her
hematocrit is 32%, leukocyte count is 800/mm3, and
platelet count is
105,000/mm3. Serum electrolyte levels are within
normal limits. Which of the
following is the most appropriate next step in

treatment?
A
) Plasmapheresis
B
) Additional chemotherapy
C
) Intravenous antibiotic therapy
D
) Intravenous corticosteroid therapy
E
) Transfusion of 2 units of leukocytes
F
) Transfusion of 2 units of packed red blood cells

20.
A 47-year-old woman who is visiting from Australia
comes to the
physician because of increasing urine output over the
past month. She has had
no dysuria or hematuria. She has a history of chronic
headaches,
peptic ulcer disease, and urinary tract infections.
An evaluation 18 months
ago for headaches, including CT scan of the head,
showed no
abnormalities; treatment with ibuprofen and phenacetin
was initiated at that time,
and her headaches have been well controlled. Her
temperature is 37.1 C
(98.8 F), blood pressure is 140/82 mm Hg, pulse is
78/min, and
respirations are 14/min. Examination shows no
abnormalities. Laboratory
studies show:
Hematocrit 32%
Mean corpuscular volume 88 m3
Serum
Glucose 130 mg/dL
Creatinine 1.7 mg/dL
Urine
Protein 2+
WBC 810/hpf
RBC none
Bacteria none
Nitrates none
Test of the stool for occult blood is negative. Which
of the following
is the most appropriate next step in management?
A

) Intravenous pyelography
B
) Discontinue current medication
C
) Antibiotic therapy for recurrent urinary tract
infections
D
) Insulin therapy for diabetes mellitus
E
) Upper endoscopy

20.
A 47-year-old woman who is visiting from Australia
comes to the
physician because of increasing urine output over the
past month. She has had
no dysuria or hematuria. She has a history of chronic
headaches,
peptic ulcer disease, and urinary tract infections.
An evaluation 18 months
ago for headaches, including CT scan of the head,
showed no
abnormalities; treatment with ibuprofen and phenacetin
was initiated at that time,
and her headaches have been well controlled. Her
temperature is 37.1 C
(98.8 F), blood pressure is 140/82 mm Hg, pulse is
78/min, and
respirations are 14/min. Examination shows no
abnormalities. Laboratory
studies show:
Hematocrit 32%
Mean corpuscular volume 88 m3
Serum
Glucose 130 mg/dL
Creatinine 1.7 mg/dL
Urine
Protein 2+
WBC 810/hpf
RBC none
Bacteria none
Nitrates none
Test of the stool for occult blood is negative. Which
of the following
is the most appropriate next step in management?
A
) Intravenous pyelography
B
) Discontinue current medication

C
) Antibiotic therapy for recurrent urinary tract
infections
D
) Insulin therapy for diabetes mellitus
E
) Upper endoscopy

21. A previously healthy 85-year-old man has had


abdominal
distention, decreased caliber of stools, and decreased
appetite over the past 2
weeks and a 9-kg (20-lb) weight loss over the past 3
months. On
sigmoidoscopy, he is found to have a constricting
adenocarcinoma of the
sigmoid colon; imaging studies show three 1-cm
metastases to the liver.
Which of the following is the most appropriate next
step in management?
A
) No treatment
B
) Radiation therapy
C
) Chemotherapy
D
) Combination radiation therapy and
chemotherapy
E
) Resection of the colon tumor
22. An 8-year-old girl with asthma is brought to
the physician 1
week after an acute exacerbation treated with a 5-day
taper course of oral
prednisone. This was her first asthma attack of the
fall season.
Medications include an inhaled corticosteroid daily
and a bronchodilator
metered-dose inhaler as needed. Her last
immunizations were at the age
of 5 years prior to entering kindergarten. Her
temperature is 37 C
(98.6 F), pulse is 92/min, and respirations are
28/min. Examination shows
end-expiratory wheezing with forced expiration.
Administration of
which of the following vaccines is most appropriate at
this visit?

A
) Haemophilus influenzae type b
B
) Influenza virus
C
) Meningococcal
D
) 23-Valent pneumococcal
E
) Varicella

23. A 27-year-old man is brought to the hospital by


family members
because he has remained in his room for 3 days. He
has refused to go to
work or eat with the family, and he expresses concern
that family or
friends may try to kill him. One week ago, he was
despondent when his
girlfriend of 5 years abruptly ended their
relationship. He has no
history of psychiatric illness. Which of the following
is the most likely
diagnosis?
A
) Adjustment disorder with depressed mood
B
) Bipolar disorder
C
) Brief psychotic disorder
D
) Dysthymic disorder
E
) Schizoaffective disorder
24. A 35-year-old primigravid woman at 20 weeks'
gestation comes to
the physician because of vaginal pressure and a
watery, pink vaginal
discharge for 1 day. Her temperature is 37.5 C (99.5
F). The uterus is
palpated at the umbilicus. Fetal heart rate is
140/min. Speculum
examination shows that the upper vagina is filled with
bulging, shiny,
smooth membranes. The cervix cannot be palpated.
Which of the following is
the most likely mechanism for these findings?
A

) Abruptio placentae
B
) Cervical incompetence
C
) Premature labor
D
) Uterine anomaly
E
) Uterine infection
25. A previously healthy 45-year-old woman has had
fever and
progressive confusion over the past 2 days. She is
now unable to perform
activities of daily living. Her temperature is 38 C
(100.4 F). She is
oriented only to person. There is no rash, and the
neck is supple. A CT
scan of the head shows normal findings. Cerebrospinal
fluid analysis
shows:
Leukocyte count 20/mm3
Glucose 45 mg/dL
Protein 110 mg/dL
Erythrocyte count
1000/mm3
Which of the following is the most likely diagnosis?
A
) Bacterial meningitis
B
) Acute alcohol intoxication
C
) Brain stem infarction
D
) Cerebral infarction
E
) Cryptococcal meningitis
F
) Enterovirus infection
G
) Hepatic encephalopathy
H
) Herpes simplex encephalitis
I
) Huntington's disease

J
) Hypoglycemia
K
) Lyme disease
26. A 57-year-old man with multiple myeloma comes
to the physician
because of a 12-hour history of fever, sharp chest
pain with deep
inspiration, and cough productive of blood-tinged
sputum. His temperature is
38.3 C (101 F), blood pressure is 120/78 mm Hg, pulse
is 112/min, and
respirations are 28/min. Crackles are heard at the
right lung base.
His hemoglobin level is 9.2 g/dL, leukocyte count is
2600/mm3, and
platelet count is 96,000/mm3. Empiric antibiotics
should be directed against
which of the following organisms?
A
) Listeria monocytogenes
B
) Neisseria meningitidis
C
) Pseudomonas aeruginosa
D
) Streptococcus bovis
E
) Streptococcus pneumoniae
27. A 19-year-old primigravid woman is brought to
the emergency
department because of a 4-hour history of heavy
vaginal bleeding. She has
vomited daily for the past month. Her last menstrual
period was 15
weeks ago. She has not received prenatal care. She
takes no medications.
Her temperature is 37 C (98.6 F), blood pressure is
140/90 mm Hg, pulse
is 80/min, and respirations are 20/min. Abdominal
examination shows a
uterus consistent in size with a 20-week gestation
with no adnexal
masses or tenderness. There is pedal edema. A serum
pregnancy test is
positive. Urinalysis shows 1+ protein. Which of the
following is the
most likely cause of this patient's vaginal bleeding?
A
) Abruptio placentae

B
) Ectopic pregnancy
C
) Hydatidiform mole
D
) Hyperthyroidism
E
) Preeclampsia
28. A 40-year-old man has the sudden onset of
excruciating head and
neck pain while carrying books from the basement to
the attic. His
temperature is 37 C (98.6 F), blood pressure is 130/90
mm Hg, and pulse is
90/min. He has photophobia and develops eye pain with
lateral eye
movements. His neck is markedly stiff and cannot be
passively flexed. He
has diffuse hyperreflexia in all extremities with
normal strength and
sensation. Plantar reflexes are flexor bilaterally.
Which of the
following is the most likely diagnosis?
A
) Cervical osteoarthritis
B
) Meningitis
C
) Ruptured cervical disc
D
) Subarachnoid hemorrhage
E
) Syringomyelia

29. A 59-year-old woman comes to the emergency


department because of
shortness of breath for 2 days. She had stage II
breast cancer 5 years
ago treated with lumpectomy, radiation, and
chemotherapy. Her
temperature is 37.5 C (99.5 F), blood pressure is
90/60 mm Hg, and respirations
are 24/min. Examination shows jugular venous
distention. Heart sounds
are distant. Which of the following is the most
appropriate next step
in management?
A
) Antibiotic therapy

B
) Anticoagulant therapy
C
) Intravenous digoxin therapy
D
) Intravenous furosemide therapy
E
) Chemotherapy
F
) Radiation therapy
G
) Pericardiocentesis
30. A 57-year-old man comes for a routine follow-up
examination. He
has a 10-year history of an intermittent facial rash.
He has been
taking propranolol for 2 months for hypertension.
Examination shows
several erythematous pustules and papules involving
the nose and central
face. There are telangiectasias at the base of the
papules. Which of the
following is the most likely explanation for these
findings?
A
) Acne rosacea
B
) Acne vulgaris
C
) Basal cell carcinoma
D
) Discoid lupus erythematosus
E
) Seborrheic dermatitis

31. A 42-year-old man comes to the physician


because of malaise,
muscle and joint pain, and temperatures to 38.4 C
(101.1 F) for 3 days.
Three months ago, he underwent cadaveric renal
transplantation resulting
in immediate kidney function. At the time of
discharge, his serum
creatinine level was 0.8 mg/dL. He is receiving
cyclosporine and
corticosteroids. Examination shows no abnormalities.
His leukocyte count is

2700/mm3, and serum creatinine level is 1.6 mg/dL;


serum cyclosporine
level is in the therapeutic range. A biopsy of the
transplanted kidney
shows intracellular inclusion bodies. Which of the
following is the most
appropriate next step in management?
A
) Increase the dosage of corticosteroids
B
) Increase the dosage of cyclosporine
C
) Begin amphotericin therapy
D
) Begin ganciclovir therapy
E
) Begin heparin therapy
32. A 5-month-old boy is brought to the physician
because of a
24-hour history of fever, cough, noisy breathing, and
difficulty feeding.
His symptoms began 3 days ago with nasal discharge,
mild cough, and
chest congestion. He appears somewhat irritable and
is crying. His
temperature is 38.5 C (101.3 F), pulse is 108/min, and
respirations are
32/min and shallow with a prolonged expiratory phase.
On examination, the
throat appears normal. A few small anterior and
posterior cervical
nodes are palpable. Both eardrums are pink but have
normal landmarks and
mobility. There is good air entry with diffuse
bilateral expiratory
wheezes on auscultation. An x-ray film of the chest
shows hyperinflation.
Which of the following is the most likely pathogen?
A
) Adenovirus
B
) Haemophilus influenzae
C
) Mycoplasma pneumoniae
D
) Respiratory syncytial virus
E
) Streptococcus pneumoniae

3. Three days after undergoing a right hip


replacement for
rheumatoid arthritis, a 77-year-old man is brought to
the physician because of a
2-day history of pain, burning, and itching of his
left eye and left
side of his forehead. He has the sensation that there
is a speck of dirt
in his left eye. Current medications include
prednisone and
methotrexate. Examination of the left eye shows
conjunctival injection and
swelling of the upper eyelid. There is an
erythematous rash over the left
side of the forehead and tenderness to palpation from
the upper eyelid
to the vertex. A photograph of the rash is shown.
Which of the
following is the most appropriate next step in
management?
A
) Measurement of erythrocyte sedimentation rate
B
) MRI of the brain with contrast
C
) Acyclovir therapy
D
) Corticosteroid therapy
E
) Lumbar puncture
34. A 54-year-old man with chronic obstructive
pulmonary disease
undergoes a total hip arthroplasty for avascular
necrosis of the femoral
head. On the second postoperative day, he has
diffuse, profound
weakness and vomiting. His blood pressure is 85/50 mm
Hg, and pulse is
100/min. The operative site is clean and dry, with
minimal output from the
drains. Hemoglobin level is 13.8 g/dL, serum sodium
level is 132 mEq/L,
and serum potassium level is 5.8 mEq/L. Which of the
following is most
likely to confirm the diagnosis?
A
) Measurement of serum thyroid-stimulating hormone
level
B
) Direct antiglobulin (Coombs') test
C
) ACTH stimulation test

D
) Ventilation-perfusion lung scans
E
) Echocardiography
35. A previously healthy 47-year-old nulliparous
woman is brought to
the emergency department by ambulance because of acute
low back pain
radiating to the right posterior leg for 2 hours. The
pain began when
she bent over at work to retrieve a file from the
lowest drawer of a
filing cabinet. She does not smoke cigarettes or
drink alcohol.
Examination shows right paraspinous muscle spasm and
pain in the lower back with
right straight-leg raising at 30 degrees. She says
that she plans to
file a claim for a work-related injury. Which of the
following findings
is the strongest risk factor for a prolonged episode
of pain in this
patient?
A
) Arrival for care in an ambulance
B
) Claim that pain is work-related
C
) Gender
D
) Nulliparity
E
) Positive straight-leg raising test
F
) Radiation of the pain into the posterior lower
extremity

36. A healthy 27-year-old woman comes to the


physician for an annual
examination. She is concerned about her risk for an
abnormal Pap smear
in the future. A history of use of which of the
following would
increase her risk for cervical cancer?
A
) Alcohol
B
) Cervical cap

C
) Cigarettes
D
) Isotretinoin
E
) IUD
37. A 72-year-old man is brought to the physician
by his daughter
because of painless jaundice for 1 month. His wife
died 10 years ago,
and his daughter is his only child. Before examining
the patient, the
daughter asks to speak privately with the physician
and asks that she be
given the results of any tests. She specifically
requests that he not
be given any "bad news." The patient is alert. His
vital signs are
within normal limits. Examination shows scleral
icterus and jaundice.
There is mild abdominal tenderness on palpation. He
is oriented to
person, place, and time. A CT scan of the abdomen
shows a pancreatic mass
with bile duct obstruction and probable metastatic
lesions in the
liver. Which of the following is the most appropriate
next step?
A
) Abide by the daughter's wishes
B
) Ask the patient if he wishes to discuss his test
results,
preferably with his daughter present
C
) Tell the daughter it is a legal requirement to
tell the patient
any and all results of medical testing
D
) Consult with the hospital attorney
E
) Ask another physician to take over the
patient's care

38. A 66-year-old man has had numbness and tingling


in the hands and
feet for 2 weeks. He lives in a homeless shelter and
is well fed. He
has been treated for pulmonary tuberculosis for 4

months with
isoniazid, rifampin, ethambutol, and pyrazinamide. He
is compliant with his
medication regimen but continues to abuse alcohol.
His temperature is 37
C (98.6 F), blood pressure is 136/76 mm Hg, pulse is
72/min, and
respirations are 20/min. He is well nourished but
depressed and irritable.
There is decreased sensation to pain and touch in the
hands and feet in
a stocking-glove distribution. Which of the following
is the most
likely nutritional deficiency?
A
) Folic acid
B
) Niacin
C
) Vitamin A
D
) Vitamin B1 (thiamine)
E
) Vitamin B2 (riboflavin)
F
) Vitamin B6
G
) Vitamin B12 (cyanocobalamin)
H
) Vitamin C
I
) Vitamin D
J
) Vitamin E
K
) Vitamin K

39. A 63-year-old man is brought to the emergency


department 3 hours
after the acute onset of severe right-sided flank
pain. He has a
9-year history of gout. His blood pressure is 110/84
mm Hg, pulse is
78/min, and respirations are 16/min. Examination
shows normal bowel sounds
and no abdominal tenderness or masses. Urinalysis
shows 40
erythrocytes/hpf. Intravenous pyelography confirms a
right ureteral calculus.

Which of the following is the most likely underlying


mechanism of this
patient's urolithiasis?
A
) An increase in urinary pH
B
) Damage to the epithelial lining of the ureters
C
) Lack of inhibitors of crystal formation
D
) Presence of urease-splitting bacteria
E
) Urinary supersaturation with uric acid
40. A 27-year-old nulligravid woman comes to the
physician for
preconceptional counseling. She has a mechanical
mitral heart valve and
chronic rheumatoid arthritis. Her cardiac status is
New York Heart
Association Class II. She feels well. Current daily
medications include
warfarin, prednisone, and acetaminophen with codeine.
Examination shows
no abnormalities except for audible clicking from the
heart valve.
Which of the following is the most appropriate advice
for this patient?
A
) Chemical dependency counseling before pregnancy
B
) Discontinuation of anticoagulant therapy during
pregnancy
C
) Discontinuation of prednisone during pregnancy
D
) Switching from warfarin to heparin before
pregnancy
E
) No change in treatment before or during pregnancy

41. A 60-year-old man comes to the physician


because of difficulty
sleeping and concentrating and a 5-kg (10-lb) weight
loss over the past
3 months. He also has become withdrawn. He has had
chronic pain since
sustaining fractures of the left lower extremity,
pelvis, and several

ribs in a motor vehicle collision 2 years ago. He has


a previous
history of alcohol abuse. He takes a nonsteroidal
anti-inflammatory drug.
Which of the following is the most appropriate
pharmacotherapy?
A
) Carbamazepine
B
) Chlordiazepoxide
C
) Disulfiram
D
) Lithium carbonate
E
) Nortriptyline
42. An 82-year-old man comes to the physician
because of a 3-day
history of low back pain that radiates to the right
leg. He also has had
a lesion over the right shin and weakness of the right
foot. He began
taking prednisone 2 weeks ago for acute bronchitis.
He has chronic
obstructive pulmonary disease, benign prostatic
hypertrophy, and glaucoma.
Examination shows numerous papular and vesicular
lesions over the right
anterior and posterior shin. There is weakness of
right knee flexion,
ankle dorsiflexion, plantar flexion, eversion, and
inversion; the right
ankle reflex is absent. Sensation to pinprick and
cold is decreased
over the right lower extremity. Which of the
following is the most
likely causal organism?
A
) Borrelia burgdorferi
B
) Epstein-Barr virus
C
) Herpes simplex virus 1
D
) Poliovirus
E
) Treponema pallidum
F
) Varicella-zoster virus

43. A 30-year-old woman comes to the physician


because of
long-standing unhappiness that may have started when
she was rejected by her
classmates as a teenager. She says that although she
has good days, many
days are dominated by negative thoughts about herself.
She appears
somewhat sad and tends to be readily critical of
herself. Although she
sleeps satisfactorily, she often finds her energy
level decreased by the
end of the day. She also has been forgetful. She
weighs 59 kg (130 lb)
and is 157 cm (62 in) tall. Physical examination and
laboratory
studies show no abnormalities. Which of the following
is the most likely
diagnosis?
A
) Adjustment disorder with depressed mood
B
) Depersonalization disorder
C
) Dissociative identity disorder
D
) Dysthymic disorder
E
) Hypothyroidism
F
) Major depressive disorder
G
) Schizoaffective disorder
For each patient with cough, select the most
appropriate next step in
diagnosis.
A
) Measurement of serum a1-antitrypsin level
B
) Methacholine challenge test
C
) Quantitative measurement of serum antibody levels
D
) Sweat chloride test

E
) Ventilation-perfusion lung scans
44. A 27-year-old woman comes to the physician
because of
intermittent episodes of shortness of breath and cough
over the past 4 months.
She says that cold weather and exercise can
precipitate her symptoms.
Examination shows no abnormalities. The lungs are
clear to auscultation
and percussion. An x-ray film of the chest and
spirometry show no
abnormalities.
A 32-year-old woman comes to the physician because of
a 4-month history
of fatigue, cough, and shortness of breath with
exertion. She has had
two episodes of pneumonia and one episode of severe
sinusitis over the
past 2 years. She has never smoked. She takes no
medications.
Crackles are heard at the left lung base. An x-ray
film of the chest shows a
left lower lobe infiltrate and scarring of the right
base.
46. A 32-year-old man is brought to the emergency
department 30
minutes after being involved in a motor vehicle
collision. He was the
restrained driver. On arrival, he is alert and has
shortness of breath.
His blood pressure is 80/50 mm Hg, pulse is 130/min,
and respirations
are 30/min. Examination shows jugular venous
distention and abrasions
over the left hemithorax. The trachea is deviated to
the right. Breath
sounds are absent on the left. Which of the following
is the most
likely cause of the hypotension?
A
) Cardiogenic shock
B
) Congestive heart failure
C
) Decreased systemic vascular resistance
D
) Decreased venous return
E
) Hypovolemic hypoperfusion
F

) Increased systemic vascular resistance


BLOCK 4:-1. A previously healthy 2-year-old boy is brought
to the physician
20 minutes after an episode of cyanosis and loss of
consciousness that
lasted 3 minutes. The symptoms occurred after his
mother scolded him
for climbing onto the dining room table. The mother
says that the child
began to cry, let out a deep sigh, stopped breathing,
and jerked his
arms and legs back and forth. On arrival, he is alert
and active.
Neurologic examination shows no focal findings. Which
of the following is
the most appropriate next step in management?
A
) Reassurance
B
) Electroencephalography
C
) CT scan of the head
D
) Anticonvulsant therapy
E
) Lumbar puncture

2. A case-control study is conducted to assess the


risk for
intussusception in infants under the age of 1 year who
receive the rotavirus
vaccine. The medical records of all those who
received the vaccine and
those who did not receive the vaccine over a 6-month
period are
reviewed. Results show 125 cases per 100,000
infant-years for infants who
received the vaccine compared to 45 cases per 100,000
infant-years for
infants who did not receive the vaccine. The
investigators conclude that
the relative risk for intussusception is 1.9 times
greater in infants
who receive the rotavirus vaccine (95% confidence
interval of 0.57.7 and
p=0.39). Which of the following is the most accurate
interpretation of
these results?
A
) The results do not show an association between
rotavirus vaccine

and intussusception, but they may be related


B
) The results show sufficient statistical power to
identify an
association between rotavirus vaccine and
intussusception
C
) Rotavirus vaccine is associated with a 39% risk
for
intussusception
D
) Rotavirus vaccine causes intussusception in 1.9%
of infants
E
) Rotavirus vaccine prevents 80 cases of
intussusception per 100,000
infant-years

3.
A 42-year-old woman, gravida 3, para 3, comes to the
emergency
department 24 hours after the onset of moderate
epigastric pain radiating to
the back. Her last menstrual period was 3 weeks ago.
She has no history
of serious illness and has never undergone an
operative procedure. She
weighs 72 kg (160 lb) and is 157 cm (62 in) tall. Her
temperature is
37.2 C (99 F), blood pressure is 130/90 mm Hg, and
pulse is 100/min.
Abdominal examination shows mild distention,
epigastric tenderness, and
voluntary guarding. Test of the stool for occult
blood is negative.
Laboratory studies show:
Hematocrit 44%
Leukocyte count 12,000/mm3
Serum
Na+ 138 mEq/L
Cl 100 mEq/L
K+ 4 mEq/L
HCO3 25 mEq/L
Bilirubin, total 1.6 mg/dL
Alkaline phosphatase 100 U/L
Aspartate aminotransferase
(AST, GOT) 14 U/L
Alanine aminotransferase
(ALT, GPT) 12 U/L
Amylase 1100 U/L
Ultrasonography shows gallstones; the gallbladder wall
is 1 mm and the

common bile duct is 5 mm in diameter. Which of the


following is the
most likely diagnosis?
A
) Acute cholecystitis
B
) Acute pancreatitis
C
) Acute perihepatitis
D
) Ascending cholangitis
E
) Duodenal ulcer
F
) Viral hepatitis
4. A 13-year-old girl is brought to the emergency
department because
of shortness of breath for 2 hours. The symptoms
began after consuming
chili, cornbread, and fruit salad with strawberries,
kiwi, and bananas.
She has a 1-year history of shortness of breath while
playing soccer or
baseball and uses a bronchodilator inhaler as needed
while exercising.
She is allergic to penicillin and pineapples. Her
blood pressure is
80/60 mm Hg, pulse is 120/min and regular, and
respirations are 20/min
with use of accessory muscles. Examination of the
lungs shows poor air
entry bilaterally with diffuse expiratory wheezes.
Which of the
following is the most appropriate initial
pharmacotherapy?
A
) Inhaled bronchodilators
B
) Inhaled cromolyn sodium
C
) Inhaled ipratropium bromide
D
) Intravenous corticosteroids
E
) Subcutaneous epinephrine
5.

A 52-year-old woman comes to the emergency department


6 days after knee
arthroplasty because of constant, right-sided chest
pain and shortness
of breath for 24 hours. Her blood pressure is 110/50
mm Hg, pulse is
114/min, and respirations are 24/min. Examination of
the heart, lungs,
and extremities shows no abnormalities. Arterial
blood gas analysis on
room air shows:
pH 7.49
PCO2 29 mm Hg
PO2 66 mm Hg
Ventilation-perfusion lung scans show a low
probability for pulmonary
embolus. An ECG shows sinus tachycardia; an x-ray
film of the chest
shows no abnormalities. After the evaluation, the
patient is pain-free
and wishes to go home. Which of the following is the
most appropriate
next step in management?
A
) Discharge home and reexamination in 2 weeks
B
) Exercise stress test
C
) Pulmonary function tests
D
) Echocardiography
E
) Pulmonary angiography
F
) Ibuprofen therapy
6. A 25-year-old woman comes to the physician
because of "spells"
characterized by sweating, palpitations, and shortness
of breath that
have awakened her at night several times over the past
3 months. She
resigned from her job as a sales clerk 6 months ago
and now works from home
as a telemarketer. She recently began going to the
grocery store late
at night because she is too nervous around people.
She says that she
has been feeling sad lately. On mental status
examination, she is fully
oriented, and her range of affect is full. Which of
the following is
the most likely diagnosis?

A
) Adjustment disorder with anxiety
B
) Adjustment disorder with depressed mood
C
) Dysthymic disorder
D
) Generalized anxiety disorder
E
) Major depressive disorder
F
) Panic disorder with agoraphobia
G
) Post-traumatic stress disorder

7. A 65-year-old woman has a 6-month history of


progressive
irritability, palpitations, heat intolerance, frequent
bowel movements, and a
6.8-kg (15-lb) weight loss. She has had a neck mass
for more than 10
years. 131I scan shows an enlarged thyroid gland with
multiple areas of
increased and decreased uptake. Which of the
following is the most
likely diagnosis?
A
) Defect in thyroxine (T4) biosynthesis
B
) Graves' disease
C
) Multinodular goiter
D
) Riedel's thyroiditis
E
) Thyroid carcinoma
F
) Thyroiditis
G
) Toxic adenoma
H
) Triiodothyronine (T3) thyrotoxicosis
A

) Acute gastrointestinal bleeding


B
) Adrenal insufficiency
C
) Aortic valve rupture
D
) Cardiac tamponade
E
) Congestive heart failure
F
) Pneumonia
G
) Pulmonary embolism
H
) Sepsis
8. A previously healthy 62-year-old man is brought
to the emergency
department by paramedics 40 minutes after the sudden
onset of severe
shortness of breath while dressing this morning. He
is unable to provide
additional medical history. He is in severe
respiratory distress. His
temperature is 37.8 C (100 F), blood pressure is 90/60
mm Hg, pulse is
120/min and regular, and respirations are 24/min.
Examination shows
marked jugular venous distention. The lungs are clear
to auscultation.
Cardiac examination shows a nondisplaced and discrete
point of maximal
impulse and normal S1 and S2; there is an S4 and a
right parasternal
heave. Abdominal examination shows no abnormalities.
There is no edema
of the lower extremities. Laboratory studies show:
Hematocrit 40%
Leukocyte count 14,000/mm3
Platelet count 350,000/mm3
Arterial blood gas analysis on 5 L/min of oxygen:
pH 7.5
PCO2 16 mm Hg
PO2 64 mm Hg
A previously healthy 67-year-old woman is brought to
the emergency

department by paramedics 40 minutes after the sudden


onset of shortness of
breath while shopping. She is unable to provide
additional medical
history. She is in severe respiratory distress. Her
temperature is 37 C
(98.6 F), blood pressure is 90/60 mm Hg, pulse is
120/min and regular,
and respirations are 24/min. Examination shows marked
jugular venous
distention. Diffuse crackles are heard throughout all
lung fields.
Cardiac examination shows an enlarged point of maximal
impulse and normal
S1 and S2; there is an S3. Abdominal examination
shows no
abnormalities. There is no edema of the lower
extremities. Laboratory studies
show:
Hematocrit 38%
Leukocyte count 12,000/mm3
Platelet count 350,000/mm3
Arterial blood gas analysis on 5 L/min of oxygen:
pH 7.5
PCO2 16 mm Hg
PO2 64 mm Hg

10. A mentally competent 76-year-old man is in the


terminal stage of
multiple myeloma. He is unable to move and requires
24-hour nursing
care. Increasing doses of narcotics are needed to
control severe pain.
As a result, when he is pain-free, respiratory
function is impaired and
consciousness is clouded. The patient says he cannot
live with this
degree of pain and asks to be given a lethal injection
of pain
medication. Which of the following is the most
appropriate step regarding the
pain medication?
A
) Reduce the dosage so as not to impair respiration
B
) Administer the dosage necessary to control pain
despite
respiratory impairment
C
) Administer the dosage necessary to control pain

and add a
centrally acting stimulant
D
) Appeal to the family to convince the patient to
tolerate a bit
more pain

11. A 2-day-old newborn is brought to the physician


because of a
generalized rash for 6 hours. The newborn is active,
alert, and feeding
well. His temperature is 36.9 C (98.4 F).
Examination shows a rash
consisting of numerous white and pale yellow papules
with a large base of
macular erythema over the trunk and extremities.
Wright's stain of
scrapings from the lesions shows eosinophils. Which
of the following is
the most appropriate next step in management?
A
) Reassurance
B
) Topical corticosteroid therapy
C
) Intravenous acyclovir therapy
D
) Intravenous ampicillin and gentamicin therapy
E
) Intravenous nafcillin therapy

12. A 4-year-old boy is brought for a well-child


examination. He
uses two-word phrases, can say his first name but not
his last name, and
cannot identify colors. He is just beginning toilet
training. His
7-year-old sister has a learning disability and
attends special education
classes. Genital development is Tanner stage 1;
testes are large.
Which of the following is the most appropriate next
step in diagnosis?
A
) Reexamination in 6 months
B
) Thyroid function tests
C

) DNA testing
D
) Measurement of bone age
E
) CT scan of the head

3. A 72-year-old man is brought to the physician


because of a 2-day
history of nausea and vomiting. The vomitus has been
clear, and no
blood has been noted. He has had a decreased appetite
for the past week.
There is no associated pain or altered bowel function.
He reports that
he is not seeing things correctly. He takes warfarin
and digoxin for
atrial fibrillation, hydrochlorothiazide for
hypertension, and potassium
supplements that he discontinued 3 weeks ago when he
ran out of
tablets. His temperature is 37 C (98.6 F), blood
pressure is 144/88 mm Hg,
and pulse is 52/min and irregular. Bowel sounds are
normal. The abdomen
is soft and nontender without rebound or guarding. No
organomegaly or
masses are palpated. Which of the following is the
most likely
diagnosis?
A
) Acute pancreatitis
B
) Brain tumor
C
) Diabetic gastroparesis
D
) Diabetic ketoacidosis
E
) Drug toxicity
F
) Food poisoning
G
) Gastric bezoar
H
) Gastric carcinoma
I
) Pyloric channel ulcer
J

) Small-bowel obstruction
K
) Uremia
14. A 27-year-old primigravid woman at 12 weeks'
gestation comes to
the emergency department 2 hours after the sudden
onset of bright red
vaginal bleeding. She has not had abdominal cramping.
Pelvic
examination shows a small amount of brownish blood in
the posterior fornix of
the vagina. The cervix is closed. The uterus is
palpable 3 cm above the
pelvic brim. Fetal heart tones are easily audible at
167/min by
Doppler. Which of the following is the most likely
diagnosis?
A
) Abruptio placentae
B
) Ectopic pregnancy
C
) Incomplete abortion
D
) Placenta previa
E
) Threatened abortion
F
) Normal pregnancy
15. A 72-year-old man is brought to the physician
by his son because
of a 4-day history of increasing confusion and memory
problems. The
son says that his father's ability to function
independently has been
generally declining over the past few years, and he
has become much more
impaired over the past week. The patient has had at
least three to four
previous episodes of a sudden decline of cognitive
functioning over the
past 3 years without full recovery. He has a history
of hypertension.
His blood pressure is 160/95 mm Hg without orthostatic
changes.
Neurologic examination shows no focal findings.
Mini-Mental State Examination
score is 21/30. Which of the following is the most
likely underlying
pathophysiologic process?

A
) Central nervous system demyelination
B
) Central nervous system infection
C
) Diffuse axonal injury
D
) Diffuse cortical atrophy
E
) Left temporal lobe infarction
F
) Multiple, small, central nervous system
infarctions
G
) Subdural hematoma

16. A 72-year-old woman comes for a routine health


maintenance
examination. She has a 3-year history of occasional
loss of small amounts
of urine when she coughs or sneezes. She has had no
pain or burning
with urination. She has hypertension treated with
daily
hydrochlorothiazide. She underwent an appendectomy at
the age of 10 years. She has
three children and had uncomplicated pregnancies.
Examination shows no
abnormalities except for a moderate cystocele. Which
of the following is
the most likely cause of this patient's urinary
symptoms?
A
) Chronic infectious trigonitis
B
) Large intravesical calculus
C
) Obstetric trauma
D
) Polycystic kidney disease
E
) Spastic neurogenic bladder
17. A 47-year-old woman is brought to the emergency
department by
her husband because of hallucinations and agitation
for 6 hours. She has
a 10-year history of alcoholism. Her last alcoholic

drink was 48 hours


ago. She is agitated and inattentive. Her
temperature is 38.3 C (101
F), blood pressure is 190/120 mm Hg, and pulse is
110/min. She is
oriented to person but not to place or time. During
the examination, she
shrieks, "Make the lizards go away." Which of the
following is the most
appropriate initial step in management?
A
) Electroencephalography
B
) CT scan of the head
C
) Intravenous ampicillin therapy
D
) Intravenous haloperidol therapy
E
) Intravenous lorazepam therapy
18. A 27-year-old woman comes to the physician
because of a 3-week
history of fever, night sweats, rash on both legs,
nonproductive cough,
and pain and swelling in her wrists and knees. She
has not had weight
loss. Her temperature is 37.7 C (99.8 F), blood
pressure is 110/70 mm
Hg, pulse is 96/min, and respirations are 14/min. The
lungs are clear
to auscultation. Cardiac examination shows no
abnormalities. There is
swelling and warmth over the wrists and knees
bilaterally and tender
red nodules on the anterior surface of both lower
extremities. An x-ray
film of the chest shows bilateral hilar fullness.
Which of the
following is the most likely diagnosis?
A
) Carcinoma of the lung
B
) Histoplasmosis
C
) Hodgkin's disease
D
) Sarcoidosis
E
) Tuberculosis

19. A 52-year-old woman comes to the physician


because of difficulty
climbing stairs for 4 months. She has also noted that
her thighs hurt
when she presses on them. She has had increasing
difficulty combing
her hair because she tires easily. On examination,
she pushes herself
out of the chair with her arms. There is weakness of
the proximal
muscles of the extremities. Which of the following is
the most likely
diagnosis?
A
) Cauda equina syndrome
B
) Cerebellar degeneration
C
) Cervical spinal cord compression
D
) Diabetic polyneuropathy
E
) Femoral artery insufficiency
F
) Guillain-Barr syndrome
G
) Lumbar spinal stenosis
H
) Multiple sclerosis
I
) Normal-pressure hydrocephalus
J
) Polymyositis
K
) Sensory neuropathy
L
) Tabes dorsalis

20. A 72-year-old man comes to the physician


because of generalized
weakness and night sweats for 6 months. During this
period he has had
a 5-kg (11-lb) weight loss. He has had polycythemia
vera for 12 years
treated with hydroxyurea and multiple phlebotomies.
Examination shows

cachexia. The liver is enlarged and nontender with a


span of 13 cm;
the spleen is enlarged. Hematocrit is 27%, leukocyte
count is 3200/mm3,
and platelet count is 150,000/mm3. A blood smear is
shown. Which of
the following is the most likely diagnosis?
A
) Acute myelogenous leukemia
B
) Cirrhosis of the liver
C
) Hodgkin's disease
D
) Miliary tuberculosis
E
) Myelofibrosis

21. A 30-year-old man has had increasingly severe


low back pain
since lifting a heavy object at work 3 days ago. The
pain does not radiate
and is not associated with bowel or bladder problems.
He has a history
of occasional stiffness and mild pain in his lower
back. There is
bilateral paravertebral muscle tenderness in the
lumbar region and limited
flexion of the lumbosacral spine. Examination of the
lower extremities
shows normal muscle strength and sensation;
straight-leg raising is
negative bilaterally. Deep tendon reflexes at the
knees and ankles are
normal. Which of the following is the most
appropriate next step in
management?
A
) Exercises to strengthen abdominal muscles
B
) Exercises to strengthen paravertebral muscles
C
) Bed rest for 5 to 7 days
D
) Use of a muscle relaxant
E
) Use of a nonsteroidal anti-inflammatory drug

22. A 42-year-old woman, gravida 2, para 2, has had


increasing
fatigue, dyspnea, orthopnea, and paroxysmal nocturnal
dyspnea over the past
2 days. She has had several episodes of hemoptysis;
she had one
episode of pulmonary edema during pregnancy 2 years
ago. A loud S1, a
snapping sound in diastole, and a rumbling diastolic
murmur are heard at the
apex. Which of the following is the most likely cause
of her
condition?
A
) Atrial myxoma
B
) Bicuspid aortic valve
C
) Postpartum cardiomyopathy
D
) Rheumatic heart disease
E
) Viral myocarditis
3. A 37-year-old woman has had unilateral breast
pain, fever, and
chills for 24 hours. She breast-feeds her 1-month-old
newborn. Her
temperature is 38 C (100.4 F). Examination shows
swelling, erythema, and
localized tenderness of the left breast. Which of the
following is the
most appropriate next step in management?
A
) Application of ice packs to the affected breast
B
) Use of a breast pump
C
) Immediate discontinuation of breast-feeding
D
) Bromocriptine therapy
E
) Penicillinase-resistant antibiotic therapy
24. A 19-year-old woman, gravida 2, para 1, at 39
weeks' gestation
is admitted in labor. Contractions occur every 2 to 3
minutes. The
cervix is 4 cm dilated and 80% effaced. She requests

an epidural for pain


control. Ten minutes after the epidural is
administered, she becomes
nauseated and diaphoretic and vomits. Her blood
pressure is 60/palpable
mm Hg. A fetal heart tracing shows sustained fetal
decelerations. The
cervix is now 8 cm dilated. The most appropriate next
step in
management is administration of which of the
following?
A
) Ephedrine
B
) Magnesium sulfate
C
) Nifedipine
D
) Oxytocin
E
) Terbutaline
25. An asymptomatic 47-year-old man comes for a
routine follow-up
examination 3 weeks after sustaining an inferior wall
myocardial
infarction. His recovery was complicated by transient
premature ventricular
contractions during the first 2 days of
hospitalization. An exercise
stress test prior to discharge showed no pain at 70%
of his predicted
maximum exercise capacity. Medications include daily
aspirin and
pravastatin. His blood pressure is 136/80 mm Hg,
pulse is 80/min and regular,
and respirations are 16/min. Cardiopulmonary
examination shows no
abnormalities. Which of the following is the most
appropriate additional
pharmacotherapy for this patient?
A
) Clonidine
B
) Metoprolol
C
) Quinidine
D
) Sulfinpyrazone
E
) Verapamil

26. A 35-year-old woman is brought to the emergency


department by
her family because of shortness of breath, tightness
in her chest, and
palpitations for 2 hours. Over the past 11 months,
she has had five
similar episodes; during the last episode 3 weeks ago,
she was treated with
an intravenous medication that caused conversion to
sinus rhythm. Her
blood pressure is 95/60 mm Hg, and pulse is 165/min
and regular. The
lungs are clear to auscultation. Which of the
following is the most
likely underlying dysrhythmia?
A
) Accelerated idioventricular rhythm
B
) Accelerated junctional rhythm
C
) Atrial fibrillation
D
) Multifocal atrial tachycardia
E
) Normal sinus rhythm
F
) Paroxysmal supraventricular tachycardia
G
) Premature supraventricular beats
H
) Premature ventricular beats
I
) Sick sinus syndrome
J
) Sinus bradycardia
K
) Sinus tachycardia
L
) Ventricular fibrillation
M
) Ventricular tachycardia

27.
A 4-year-old boy is brought to the emergency
department because of a
1-day history of fever and increasing difficulty
breathing. Over the
past year, he has had recurrent bacterial infections
including cervical
lymphadenitis, septic arthritis, and pneumonia. His
temperature is 38.9
C (102 F), blood pressure is 80/60 mm Hg, pulse is
115/min, and
respirations are 38/min and labored. Breath sounds
are decreased over the
left anterior chest. Scattered crackles are heard on
auscultation. A
thoracentesis shows purulent fluid. Laboratory
studies show:
Hematocrit 36%
Leukocyte count 18,000/mm3
Segmented neutrophils 85%
Lymphocytes 15%
Platelet count 200,000/mm3
Pleural fluid
Leukocyte count 75,000/mm3
Segmented neutrophils 98%
Lymphocytes 2%
Nitroblue tetrazolium test is abnormal. A Gram's
stain of the pleural
fluid shows numerous gram-positive cocci in the
segmented neutrophils.
An x-ray film of the chest shows left lower lobe
pneumonia with pleural
effusion. The most likely cause of this patient's
symptoms is a defect
of which of the following?
A
) Chemotaxis
B
) Immotile cilia
C
) Opsonization
D
) Phagocytic oxidative metabolism
E
) Phagocytosis
F
) T-lymphocyte function

28. A 59-year-old woman comes to the emergency


department 45 minutes

after the onset of chest discomfort that is not


relieved by three
nitroglycerin tablets. Over the past 3 months, she
has had similar episodes
characterized by nonradiating pain and a feeling of
heaviness; the
episodes were exacerbated by exertion or heavy meals
and were slowly
relieved by rest. Sublingual nitroglycerin has
provided rapid relief of
symptoms in the past. She has hypercholesterolemia,
type 2 diabetes
mellitus, and peptic ulcer disease. She smoked two
packs of cigarettes daily
for 25 years but quit 5 years ago. She appears
anxious and diaphoretic
and is nauseated. Examination shows no other
abnormalities except for
an S4. Which of the following is the most likely
diagnosis?
A
) Acute aortic dissection
B
) Angina pectoris
C
) Esophageal spasm
D
) Myocardial infarction
E
) Pulmonary embolism

29. A 27-year-old woman comes to the physician


because of
increasingly severe daily headaches over the past 3
months. The headaches are
diffuse and often occur at the base of the skull. For
the past month,
she has had brief episodes of visual darkening when
standing. She has
been amenorrheic for the past year and has had an
18-kg (40-lb) weight
gain during this period. She now weighs 118 kg (260
lb) and is 152 cm
(60 in) tall. Funduscopic examination shows
papilledema, several flame
hemorrhages, and an enlarged blind spot bilaterally.
Visual acuity is
20/20 bilaterally. A CT scan of the head shows no
abnormalities. Which
of the following is the most likely underlying cause
of these findings?
A
) Cerebral venous occlusion

B
) Communicating hydrocephalus
C
) Cytotoxic edema
D
) Idiopathic intracranial hypertension
E
) Impaired absorption of cerebrospinal fluid
F
) Infratentorial mass lesion
G
) Interstitial edema
H
) Overproduction of cerebrospinal fluid
I
) Vasogenic edema

30. A 75-year-old man with a 3-year history of


progressive cognitive
impairment due to dementia, Alzheimer's type, has had
nocturnal
disorientation for 2 weeks. He lives at home with his
wife. He is otherwise
healthy and takes no medications. Physical
examination shows normal
findings. He is disoriented to time and place, has
poor short-term
memory, is unable to do simple arithmetic, and has a
poor understanding of
general information. Which of the following is the
most appropriate
initial step in management?
A
) Increase in home nighttime lighting
B
) Prescription for chloral
ydrate
C
) Prescription for diazepam
D
) Prescription for haloperidol
E
) Use of nighttime mechanical restraints

31. A 62-year-old woman comes to the physician


because of severe

pain and swelling of her right knee for 1 day. She


has no history of
joint disease or trauma to the knee. She has
hypertension treated with
hydrochlorothiazide and type 2 diabetes mellitus
treated with glyburide.
She is sexually active only with her husband, and they
have sexual
intercourse one to two times each week. Her
temperature is 37 C (98.6 F),
blood pressure is 140/84 mm Hg, and pulse is 80/min.
Examination of
the right knee shows edema, erythema, and exquisite
tenderness to light
touch; there is an effusion. The remainder of the
examination shows no
abnormalities. Which of the following is the most
likely mechanism of
these findings?
A
) Immune complex deposition
B
) Inflammatory reaction to antisynovial antibodies
C
) Inflammatory reaction to monosodium urate
crystals
D
) Neisseria gonorrhoeae infection
E
) Streptococcus pneumoniae infection
32. Two days after a cholecystectomy, a 42-year-old
woman has
shortness of breath. Her temperature is 37.5 C (99.5
F), blood pressure is
110/70 mm Hg, pulse is 103/min, and respirations are
24/min. There is
abdominal tenderness. Examination shows no other
abnormalities. An
x-ray film of the chest shows minimal linear markings
in the right lower
lobe. Arterial blood gas analysis on 45% oxygen
shows:
pH 7.41
PCO2 40 mm Hg
PO2 52 mm Hg
Which of the following is the most likely explanation
for her hypoxia?
A
) Acute respiratory distress syndrome
B
) Atelectasis

C
) Congestive heart failure
D
) Fat embolism syndrome
E
) Pneumonia
F
) Pneumothorax

33. A 37-year-old woman comes to the emergency


department because of
a 3-day history of increasingly severe abdominal pain,
nausea, and
vomiting. Twelve years ago, she had a hysterectomy
because of severe
dysfunctional uterine bleeding. Her temperature is 37
C (98.6 F), blood
pressure is 106/70 mm Hg, pulse is 110/min, and
respirations are 12/min.
Examination shows a distended, tympanic abdomen with
diffuse tenderness
but no guarding; bowel sounds are hypoactive. Her
leukocyte count is
10,000/mm3, and hematocrit is 44%. An x-ray film of
the abdomen is
shown. Which of the following is the most appropriate
initial step in
management?
A
) CT scan of the abdomen
B
) Intravenous neostigmine therapy
C
) Esophagogastroduodenoscopy
D
) Nasogastric intubation
E
) Laparotomy

34. A healthy 27-year-old woman comes for a routine


health
maintenance examination. Her blood pressure is 185/90
mm Hg. Examination shows
no other abnormalities except for hypertensive
retinopathy. Serum
studies show a sodium level of 140 mEq/L, potassium
level of 4 mEq/L, and
creatinine level of 1.1 mg/dL. A complete blood
count, serum

catecholamine levels, and urinalysis are within normal


limits. At two subsequent
visits, her blood pressure is 190/100 mm Hg and 182/96
mm Hg,
respectively. Which of the following is the most
appropriate next step in
diagnosis?
A
) 24-Hour urine collection for measurement of
creatinine clearance
B
) 24-Hour urine collection for measurement of
17-hydroxycorticosteroid and total 17-ketosteroid
levels
C
) Measurement of serum aldosterone level
D
) Magnetic resonance angiography of renal vessels
E
) CT scan of the abdomen

35. A 76-year-old man has had fatigue and loss of


interest in daily
activities over the past 4 months. He sleeps poorly
and has had a
4.5-kg (10-lb) weight loss during this period. He
states that he has
probably lived long enough. His blood pressure is
110/78 mm Hg, and pulse
is 68/min. Examination shows a slow return of deep
tendon reflexes.
Measurement of which of the following serum levels is
the most
appropriate next step in management?
A
) Calcium
B
) Creatinine
C
) Glucose
D
) Testosterone
E
) Thyroid-stimulating hormone
36. A 29-year-old woman is brought to the physician
by her father.
She has been working continuously without sleep for 3
days on a project

which she claims "Allah and Jesus have told me to do"


that will "unify
all knowledge in physics and medicine and philosophy."
She hears the
voice of her dead brother telling her that she will be
"the next Virgin
Mary." Her father states that she was treated for
depression for 1 year
at the age of 19 years. Which of the following is the
most likely
diagnosis?
A
) Bipolar disorder
B
) Borderline personality disorder
C
) Major depressive disorder
D
) Schizophrenia
E
) Substance-induced mood disorder

37. A 67-year-old man is hospitalized for treatment


of renal
insufficiency. Three days after admission, his pulse
is 40/min. An ECG shows
tall, tented T waves. Serum studies show a sodium
level of 134 mEq/L,
potassium level of 6.9 mEq/L, and glucose level of 85
mg/dL. The most
appropriate next step in management is intravenous
administration of
which of the following?
A
) Calcium, furosemide, and 3% saline
B
) Calcium, insulin, and digitalis
C
) Calcium, insulin, and glucose
D
) Glucose, furosemide, and phosphate
E
) Glucose, glucagon, and bicarbonate
38. Six months after the delivery of her fourth
child, a 37-year-old
woman undergoes laparoscopic tubal ligation. Menses
occur at regular
28-day intervals. During the operation, she is found

to have a small
dark lesion in the cul-de-sac and filmy adhesions
surrounding the
ovaries. A biopsy specimen of a cul-de-sac lesion
confirms the diagnosis of
endometriosis. Which of the following is the most
appropriate next step
in management?
A
) Danazol therapy
B
) Gonadotropin-releasing hormone agonist therapy
C
) Oral contraceptive therapy
D
) Total abdominal hysterectomy and bilateral
salpingo-oophorectomy
E
) No further treatment indicated
A
) Administration of parenteral antibiotics
B
) Admission to the hospital for medical management
C
) Admission to the hospital for operative
management
D
) Colon contrast studies
E
) Discharge for follow-up by personal physician
F
) Endoscopy
G
) MRI of the abdomen
H
) Observation in the emergency department
39. A previously healthy 6-month-old boy is brought
to the emergency
department because of a 12-hour history of
intermittent episodes of
inconsolable crying associated with drawing up of the
legs. Over the past
6 hours, he has had intermittent diarrhea that is
reddish and mucoid,
and for the past 3 hours he has been somnolent. On
examination, he is
sleepy but arousable. His temperature is 38.1 C

(100.6 F), blood


pressure is 90/55 mm Hg, pulse is 140/min, and
respirations are 38/min. He
cries when his abdomen is palpated; a mass is felt in
the right lower
quadrant. His leukocyte count is 12,400/mm3 (50%
segmented neutrophils,
8% bands, 1% eosinophils, 40% lymphocytes, and 1%
monocytes). Serum
electrolyte levels are within normal limits. An x-ray
film of the
abdomen shows no free air.
A
) Administration of parenteral antibiotics
B
) Admission to the hospital for medical management
C
) Admission to the hospital for operative
management
D
) Colon contrast studies
E
) Discharge for follow-up by personal physician
F
) Endoscopy
G
) MRI of the abdomen
H
) Observation in the emergency department
40. A previously healthy 14-year-old girl is
brought to the
emergency department because of abdominal pain for 12
hours. She has a 1-week
history of brownish vaginal discharge. Menarche was
at the age of 12
years, and her periods have occurred at regular 28-day
intervals over
the past year. Her last menstrual period was 7 weeks
ago. Her
temperature is 37 C (98.6 F), blood pressure is 85/55
mm Hg, pulse is 145/min,
and respirations are 24/min. Abdominal examination
shows generalized
tenderness, and there is guarding with rebound in the
right lower
quadrant. Her hematocrit is 24%, and leukocyte count
is 9400/mm3 (60%
segmented neutrophils, 3% bands, 1% eosinophils, 35%
lymphocytes, and 1%
monocytes). Serum electrolyte levels are within
normal limits.

41. A 49-year-old woman is admitted to the hospital


because of renal
failure. She has had episodes of flank pain over the
past 20 years.
She has also had nocturia 2 to 3 times nightly for 10
years. Her blood
pressure is 160/100 mm Hg. Examination shows pale
mucous membranes. A
mass is palpated in the right flank. Which of the
following is the
most likely diagnosis?
A
) Horseshoe kidney
B
) Nephrolithiasis
C
) Papillary necrosis
D
) Polycystic kidney disease
E
) Renal cell carcinoma
42. On routine annual screening, an asymptomatic
27-year-old man has
a positive PPD skin test. One year ago, a PPD skin
test was negative.
He works as a nurse. Three years ago, he was
diagnosed with hepatitis
A after a trip to South America. Examination shows no
abnormalities.
His serum aspartate aminotransferase (AST, GOT)
activity is 10 U/L, and
serum alanine aminotransferase (ALT, GPT) activity is
14 U/L. An x-ray
film of the chest shows no abnormalities. Which of
the following is
the most appropriate chemoprophylaxis?
A
) Isoniazid and folic acid supplementation
B
) Isoniazid and rifampin
C
) Isoniazid and vitamin B1 (thiamine)
supplementation
D
) Isoniazid and vitamin B6 supplementation
E
) No prophylaxis indicated

43.
A 4-month-old boy is brought to the physician because
of a 2-day
history of fever and progressive redness around his
right eye. He has had
persistent diarrhea and oral candidiasis since birth
and was treated for
pneumococcal pneumonia at the age of 2 months. He
appears ill. His
temperature is 39 C (102.2 F), pulse is 130/min, and
respirations are
25/min. Examination shows violaceous preseptal
(periorbital) cellulitis
and oral candidiasis. Laboratory studies show:
Hemoglobin 10 g/dL
Leukocyte count 3000/mm3
Segmented neutrophils 85%
Lymphocytes 15%
Platelet count 350,000/mm3
Serum
IgA <5 mg/dL
IgG 300 mg/dL
IgM <5 mg/dL
Which of the following is the most likely diagnosis?
A
) AIDS
B
) Chronic granulomatous disease
C
) Severe combined immunodeficiency
D
) Thymic-parathyroid dysplasia (DiGeorge syndrome)
E
) X-linked agammaglobulinemia

44. One week after undergoing an uncomplicated


liver transplant for
biliary atresia, a 3-year-old boy appears jaundiced.
Examination shows
scleral icterus. His serum aspartate aminotransferase
(AST, GOT)
activity has increased to 1300 U/L, and serum alanine
aminotransferase (ALT,
GPT) activity has increased to 2500 U/L. His serum
bilirubin level is
3.5 mg/dL, and serum alkaline phosphatase activity is
100 U/L. Which
of the following is the most likely artery responsible
for this
patient's gastrointestinal symptoms?

A
) Hepatic
B
) Ileocolic
C
) Inferior mesenteric
D
) Left gastric
E
) Left gastroepiploic
F
) Middle colic
G
) Posterior penetrating
H
) Right colic
I
) Right gastroepiploic
J
) Splenic
K
) Superior hemorrhoidal
L
) Superior mesenteric

45. A 2-year-old girl has had fever and bloody


diarrhea for 10 days.
A stool culture obtained 7 days ago grew Salmonella
species sensitive
to amoxicillin. A blood culture was negative.
Despite beginning oral
amoxicillin therapy 4 days ago, her diarrhea has
persisted. Current
examination shows no other abnormalities except for a
temperature of 38.6 C
(101.5 F). Which of the following is the most likely
explanation for
the failure of amoxicillin to improve her symptoms?
A
) Amoxicillin does not alter the course of
Salmonella enteritidis
B
) Amoxicillin has caused pseudomembranous colitis
C
) Amoxicillin is absorbed at the level of the
jejunum, leaving no

drug to be delivered to the colon


D
) Oral amoxicillin is not absorbed into the
systemic circulation in
the presence of diarrhea
E
) Salmonella has expressed an inducible -lactamase
that inactivates
amoxicillin
46.
A 23-year-old man comes to the physician because of a
1-month history
of intermittent right-sided abdominal pain, nausea,
and vomiting. He
has sickle cell disease and has been treated several
times for painful
crises. Examination of the abdomen shows tenderness
to palpation of the
right upper quadrant on inspiration. Laboratory
studies show:
Hematocrit 25%
Leukocyte count 11,000/mm3
Serum
Bilirubin
Total 3.2 mg/dL
Direct 0.3 mg/dL
Alkaline phosphatase 56 U/L
Ultrasonography of the gallbladder shows a filling
defect. Which of
the following is the most likely cause of this
patient's
hyperbilirubinemia?
A
) Aggregation of cholesterol in the gallbladder
B
) Inhibition of glucuronosyltransferase
C
) Lysis of erythrocytes
D
) Malnutrition-induced cirrhosis
E
) Neoplastic growth in the gallbladder
BLOCK 1
1. 1. e
2. a
3. B

4. E
5. a? b?
6. b
7. b
8. C
9. D
10. A
11. J
12. H
13. d
14. a
15. d
16. b
17.
18. d
19. A
20. c
21. A
22. B
23. a
24. b
25. B/D
26. A
27. e
28. C
29. D
30. G
31. D
32. B
33. E
34. A
35. c
36. a
37. f
38.A
39. G
40. a
41. E
42. B
43. E
44. D
45. B
46. F
BLOCK 2
1 .D
2.B
3.A
4. G??
5. C
6. C
7.B
8.D
9.A
10.C??
11.C
12.D
13. C
14.C
15.A
16.C
17. A /?? E

18.B
19.B
20.E
21.E
22.D
23.D
24. E
25.C
26.F
27.B
28.E
29.B
30.D
31.E
32.B
33.A
34.A
35.E
36.E
37.E
38.F
39.D
40.B ??
41.E
42.D
43.E
44. E
45. C( OSTEOMYLITIS)
46.D
BLOCK 3
1.d
2.c
3.c
4.b
5.a
6.a
7.b
8.d
9.c
10.b
11.e
12.b
13.d
14.a
16.e
17.c
18.g
19.c
20.b
21.e
22.b
23.c
24.b
25.h
26.e
27.c
28.d?
29.g
30.a

31.d.
32.d
33.c
34.c
35.b
36.c.
37.b
38.d
39.a
40.d
41.e.
42.f
43.d
44.b
45.c
46.d
BLOCK 4
1.a
2.a
3.b?
4. e/a
5.e/b
6.f
7.c
8.c/g
9.g
10.a/c.. very controvertial plz explain
11.a
12.c
13.e
14.e
15.f
16.c
17.e/d
18.d
19.j
20.a /? E
21.e/d
22.d
23.a
24.a?
25.b
26.f
27.d
28.d
29.d
30.a
31.b
32.b
33.d nosogastric suction
34.d
35.e
36.a
37.b
38e?
39.d
40.d
41.d
42.d
43.c

44.a
45.b??
46.c

You might also like